+7 495 120-13-73 | 8 800 500-97-74

(для регионов бесплатно)

Содержание

Сечение кабеля (провода) по току и мощности таблица

При прокладке электропроводки в частном доме или квартире важно правильно подобрать сечение используемых проводов (кабелей). Если взять слишком толстый кабель (большого сечения) — это «влетит вам в копеечку», так как его цена сильно зависит от диаметра токопроводящих жил. Применение же тонкого кабеля, приводит к его перегрузке и, при несрабатывании защиты, перегреву, оплавлению изоляции, короткому замыканию и пожару. Правильным будет выбор сечения провода в зависимости от тока, что отражено в приведенных ниже таблицах.

Сечение кабеля

Сечение кабеля — это площадь среза токоведущей жилы. Если срез жилы круглый (как в большинстве случаев) и состоит из одной проволочки — то площадь/сечение определяется по формуле площади круга. Если в жиле много проволочек, то сечением будет сумма сечений всех проволочек в данной жиле.

Величины сечения во всех странах стандартизированы, причем стандарты бывшего СНГ и Европы в этой части полностью совпадают. В нашей стране документом, которым регулируется этот вопрос, являются «Правила устройства электроустановок» или кратко — ПУЭ.

Сечение кабеля выбирается исходя из нагрузок с помощью специальных таблиц, называемых «Допустимые токовые нагрузки на кабель.» Если нет никакого желания разбираться в этих таблицах — то Вам вполне достаточно знать, что на розетки желательно брать медный кабель сечением 1,5-2,5 мм², а на освещение — 1,0-1,5мм².

Для ввода одной фазы в рядовую 2-3 комнатную квартиру вполне хватит 6,0 мм². Все равно на Ваших 40-80 м² большего оборудования не поместиться, даже с учетом электроплиты.

Многие электрики для «прикидки» нужного сечения считают, что 1 мм² медного провода может пропустить через себя 10А электрического тока: соответственно 2,5 мм² меди способны пропустить 25А, а 4,0 мм² — 40А и т.д. Если Вы немного проанализируете таблицу выбора сечения кабеля, то увидите, что такой метод годится только для прикидки и только для кабелей сечением не выше 6,0 мм².

Ниже дана сокращенная таблица выбора сечения кабеля до 35 мм² в зависимости от токовых нагрузок. Там же для Вашего удобства приведена суммарная мощность электрооборудования при 1-фазном (220В) и 3-фазном (380В) потреблении.

При прокладке кабеля в трубе (т.е. в любых закрытых пространствах) возможные токовые нагрузки на кабель должны быть меньше, чем при прокладке открыто. Это связано с тем, что кабель в процессе эксплуатации нагревается, а теплоотдача в стене или в земле значительно ниже, чем на открытом пространстве.

Когда нагрузка называется в кВт — то речь идет о совокупной нагрузке. Т.е. для однофазного потребителя нагрузка будет указана по одной фазе, а для трехфазного — совокупно по всем трем. Когда величина нагрузки названа в амперах (А) — речь всегда идет о нагрузке на одну жилу (или фазу).

Таблица нагрузок по сечению кабеля:

Сечение кабеля, мм²Проложенные открытоПроложенные в трубе
медьалюминиймедьалюминий
ток, Амощность, кВтток, Амощность, кВтток, Амощность, кВтток, Амощность, кВт
220В380В220В380В220В380В220В380В
0. 5112.4
0.75153.3
1173.76.41435.3
1.52358.7153.35.7
2.5306.611245.29.1214.67.9163.56
44191532712275.910214.67.9
6501119398.514347.412265.79.8
10801730601322501119388. 314
161002238751628801730551220
25140305310523391002238651424
35170376413028491352951751628

Для самостоятельного расчета необходимого сечение кабеля, например, для ввода в дом, можно воспользоваться кабельным калькулятором или выбрать необходимое сечение по таблице.

Настоящая таблица касается кабелей и проводов в резиновой и пластмассовой изоляции. Это такие широко распространенные марки как: ПВС, ВВП, ВПП, ППВ, АППВ, ВВГ. АВВГ и ряд других. На кабели в бумажной изоляции есть своя таблица, на не изолированные провода и шины — своя.

При расчетах сечения кабеля специалист должен также учитывать методы прокладки кабеля: в лотках, пучками и т. п.

    Кроме того, величины из таблиц о допустимых токовых нагрузках должны быть откорректированы следующими снижающими коэффициентами:
  • поправочный коэффициент, соответствующий сечению кабеля и расположению его в блоке;
  • поправочный коэффициент на температуру окружающей среды;
  • поправочный коэффициент для кабелей, прокладываемых в земле;
  • поправочный коэффициент на различное число работающих кабелей, проложенных рядом.

Расчет сечения провода

Начнем не с таблицы, а с расчета. То есть, каждый человек, не имея под рукой интернет, где в свободном доступе ПУЭ с таблицами имеется, может самостоятельно определить сечение кабеля по току. Для этого потребуется штангенциркуль и формула.

Если рассмотреть сечение кабеля, то это круг с определенным диаметром.
Существует формула площади круга: S= 3,14*D²/4, где 3,14 – это Архимедово число, «D» — диаметр измеренной жилы. Формулу можно упростить: S=0,785*D².

Если провод состоит из нескольких жил, то замеряется диаметр каждой, вычисляется площадь, затем все показатели суммируются. А как вычислить сечение кабеля, если каждая его жила состоит из нескольких тоненьких проводков?

Процесс немного усложняется, но не сильно. Для этого придется подсчитать количество проводков в одной жиле, измерить диаметр одного проводка, вычислить его площадь по описанной формуле и умножить данный показатель на количество проводков. Это и будет сечение одной жилы. Теперь необходимо это значение умножить на количество жил.

Если нет желания считать проводки и измерять их размеры, надо просто замерить диаметр одной жилы, состоящий из нескольких проводов. Снимать размеры надо аккуратно, чтобы не смять жилу. Обратите внимание, что этот диаметр не является точным, потому что между проводками остается пространство.

Соотношение тока и сечения

Чтобы понять, как работает электрический кабель, необходимо вспомнить обычную водопроводную трубу. Чем больше ее диаметр, тем больше воды через нее будет проходить. То же самое и с проводами.

Чем больше их площадь, тем большей силы ток, через них пройдет, тем большую нагрузку такой провод выдерживает. При этом кабель не будет перегреваться, что является самым важным требованием правил пожарной безопасности.

Поэтому связка сечение – ток является основным критерием, который используется в подборе электрических проводов в разводке. Поэтому вам необходимо сначала разобраться, сколько бытовых приборов и какой общей мощности будет подключены к каждому шлейфу.

Сечение жилы провода, мм2Медные жилыАлюминиевые жилы
Ток, АМощность, ВтТок, АМощность, Вт
0.561300
0.75102200
1143100
1.5153300102200
2194200143100
2.5214600163500
4275900214600
6347500265700
105011000388400
1680176005512100
25100220006514300

К примеру, на кухне обязательно устанавливается холодильник, микроволновка, кофемолка и кофеварка, электрочайник иногда посудомоечная машина. То есть, все эти прибору могут в один момент быть включены одновременно. Поэтому в расчетах и используется суммарная мощность помещения.

Узнать потребляемую мощность каждого прибора можно из паспорта изделия или на бирке.

    Для примера обозначим некоторые из них:
  1. Чайник – 1-2 кВт.
  2. Микроволновка и мясорубка 1,5-2,2 кВт.
  3. Кофемолка и кофеварка – 0,5-1,5 кВт.
  4. Холодильник 0,8 кВт.

Узнав мощность, которая будет действовать на проводку, можно подобрать ее сечение из таблицы. Не будем рассматривать все показатели данной таблицы, покажем те, которые преобладают в быту.

Чем отличается кабель от провода

Прежде чем перейти к основному содержимому, нам необходимо понять, что же мы все-таки хотим рассчитать, сечение провода или кабеля, в чем различия одного от другого!? Несмотря на то, что обыватель применяет эти два слова как синонимы, подразумевая под этим что-то свое, но если быть дотошными, то разница все же имеется.

Так провод это одна токопроводящая жила, будь то моножила или набор проводников, изолированная в диэлектрик, в оболочку. А вот кабель, это уже несколько таких проводов, объединенных в единое целое, в своей защитной и изоляционной оболочке. Для того, чтобы вам было лучше понятно, что к чему, взгляните на картинку.

Так вот, теперь мы в курсе, что рассчитывать нам необходимо именно сечение провода, то есть одного токопроводящего элемента, а второй будет уже уходить от нагрузки, обратно к питанию.

Однако мы порой и сами забываемся не лучше Вашего, так что если вы нас подловите на том, что где-то все же встретится слово кабель, то не сочтите уж за невежество, стереотипы делают свое дело.

Выбор кабеля

Делать внутреннюю разводку лучше всего из медных проводов. Хотя алюминиевые им не уступят. Но тут есть один нюанс, который связан с правильно проведенном соединении участков в распределительной коробке. Как показывает практика, места соединений часто выходят из строя из-за окисления алюминиевого провода.

Еще один вопрос, какой провод выбрать: одножильный или многожильный? Одножильный имеет лучшую проводимость тока, поэтому именно его рекомендуют к применению в бытовой электрической разводке. Многожильный имеет высокую гибкость, что позволяет его сгибать в одном месте по несколько раз без ущерба качеству.

Одножильный или многожильный

При монтаже электропроводки обычно применяют провода и кабели марки ПВС, ВВГнг, ППВ, АППВ. В этом списке встречаются как гибкие кабели, так и с моножилой.

Здесь мы хотели бы сказать вам одну вещь. Если ваша проводка не будет шевелиться, то есть это не удлинитель, не место сгиба, которое постоянно меняет свое положение, то предпочтительно использовать моножилу.

Вы спросите почему? Все просто! Не смотря на то, насколько хорошо не были бы уложены в защитную изоляционною оплетку проводники, под нее все же попадет воздух, в котором содержится кислород. Происходит окисление поверхности меди.

В итоге, если проводников много, то площадь окисления намного больше, а значит токопроводящее сечение «тает» на много больше. Да, это процесс длительный, но и мы не думаем, что вы собрались менять проводку часто. Чем больше она проработает, тем лучше.

Особенно это эффект окисления будет сильно проявляться у краев реза кабеля, в помещениях с перепадом температуры и при повышенной влажности. Так что мы вам настоятельно рекомендуем использовать моножилу! Сечение моножилы кабеля или провода изменится со временем незначительно, а это так важно, при наших дальнейших расчетах.

Медь или алюминий

В СССР большинство жилых домов оснащались алюминиевой проводкой, это было своеобразной нормой, стандартом и даже догмой. Нет, это совсем не значит, что страна была бедная, и не хватало на меди. Даже в некоторых случая наоборот.

Но видимо проектировщики электрических сетей решили, что экономически можно много сэкономить, если применять алюминий, а не медь. Действительно, темпы строительства были огромнейшие, достаточно вспомнить хрущевки, в которых все еще живет половина страны, а значит эффект от такой экономии был значительным. В этом можно не сомневаться.

Тем не менее, сегодня другие реалии, и алюминиевую проводку в новых жилых помещениях не применяют, только медную. Это исходит из норм ПУЭ пункт 7.1.34 «В зданиях следует применять кабели и провода с медными жилами…».

Так вот, мы вам настоятельно не рекомендуем экспериментировать и пробовать алюминий. Минусы его очевидны. Алюминиевые скрутки невозможно пропаять, так же очень трудно сварить, в итоге контакты в распределительных коробках могут со временем нарушиться. Алюминий очень хрупкий, два-три изгиба и провод отпал.

Будут постоянные проблемы с подключением его к розеткам, выключателем. Опять же если говорить о проводимой мощности, то медный провод с тем же сечением для алюминия 2,5 мм.кв. допускает длительный ток в 19А, а для меди в 25А. Здесь разница больше чем 1 КВт.

Так что еще раз повторимся — только медь! Далее мы и будем уже исходить из того, что сечение рассчитываем для медного провода, но в таблицах приведем значения и для алюминия. Мало ли что.

Зачем производится расчет

Провода и кабели, по которым протекает электрический ток, являются важнейшей частью электропроводки.

Расчет сечения провода необходимо производить затем, чтобы убедится, что выбранный провод соответствует всем требованиям надежности и безопасной эксплуатации электропроводки.

Безопасная эксплуатация заключается в том, что если вы выберете сечение, не соответствующее его токовым нагрузкам, то это приведет к чрезмерному перегреву провода, плавлению изоляции, короткому замыканию и пожару.

Поэтому к вопросу о выборе сечения провода необходимо отнестись очень серьезно.

Что нужно знать

Основным показателем, по которому рассчитывают провод, является его длительно допустимая токовая нагрузка. Проще говоря, это такая величина тока, которую он способен пропускать на протяжении длительного времени.

Чтобы найти величину номинального тока, необходимо подсчитать мощность всех подключаемых электроприборов в доме. Рассмотрим пример расчета сечения провода для обычной двухкомнатной квартиры.

Таблица потребляемой мощности/силы тока бытовыми электроприборами


ЭлектроприборПотребляемая мощность, ВтСила тока, А
Стиральная машина2000 – 25009,0 – 11,4
Джакузи2000 – 25009,0 – 11,4
Электроподогрев пола800 – 14003,6 – 6,4
Стационарная электрическая плита4500 – 850020,5 – 38,6
СВЧ печь900 – 13004,1 – 5,9
Посудомоечная машина2000 – 25009,0 – 11,4
Морозильники, холодильники140 – 3000,6 – 1,4
Мясорубка с электроприводом1100 – 12005,0 – 5,5
Электрочайник1850 – 20008,4 – 9,0
Электрическая кофеварка630 – 12003,0 – 5,5
Соковыжималка240 – 3601,1 – 1,6
Тостер640 – 11002,9 – 5,0
Миксер250 – 4001,1 – 1,8
Фен400 – 16001,8 – 7,3
Утюг900 –17004,1 – 7,7
Пылесос680 – 14003,1 – 6,4
Вентилятор250 – 4001,0 – 1,8
Телевизор125 – 1800,6 – 0,8
Радиоаппаратура70 – 1000,3 – 0,5
Приборы освещения20 – 1000,1 – 0,4

После того как мощность будет известна расчет сечения провода или кабеля сводится к определению силы тока на основании этой мощности. Найти силу тока можно по формуле:

1) Формула расчета силы тока для однофазной сети 220 В:

расчет силы тока для однофазной сети

где Р — суммарная мощность всех электроприборов, Вт;
U — напряжение сети, В;
КИ= 0.75 — коэффициент одновременности;
cos для бытовых электроприборов- для бытовых электроприборов.
2) Формула для расчета силы тока в трехфазной сети 380 В:

расчет силы тока для трехфазной сети

Зная величину тока, сечение провода находят по таблице. Если окажется что расчетное и табличное значения токов не совпадают, то в этом случае выбирают ближайшее большее значение. Например, расчетное значение тока составляет 23 А, выбираем по таблице ближайшее большее 27 А — с сечением 2.5 мм2.

Какой провод лучше использовать

На сегодняшний день для монтажа, как открытой электропроводки, так и скрытой, конечно же большой популярностью пользуются медные провода.

    Медь, по сравнению с алюминием, более эффективна:
  • она прочнее, более мягкая и в местах перегиба не ломается по сравнению с алюминием;
  • меньше подвержена коррозии и окислению. Соединяя алюминий в распределительной коробке, места скрутки со временем окисляются, это приводит к потере контакта;
  • проводимость меди выше чем алюминия, при одинаковом сечении медный провод способен выдержать большую токовую нагрузку чем алюминиевый.

Недостатком медных проводов является их высокая стоимость. Стоимость их в 3-4 раза выше алюминиевых. Хотя медные провода по стоимости дороже все же они являются более распространенными и популярными в использовании чем алюминиевые.

Расчет сечения медных проводов и кабелей

Подсчитав нагрузку и определившись с материалом (медь), рассмотрим пример расчета сечения проводов для отдельных групп потребителей, на примере двухкомнатной квартиры.

Как известно, вся нагрузка делится на две группы: силовую и осветительную.

В нашем случае основной силовой нагрузкой будет розеточная группа, установленная на кухне и в ванной. Так как там устанавливается наиболее мощная техника (электрочайник, микроволновка, холодильник, бойлер, стиральная машина и т. п.).

Для этой розеточной группы выбираем провод сечением 2.5 мм2. При условии, что силовая нагрузка будет разбросана по разным розеткам. Что это значит? Например, на кухне для подключения всей бытовой техники нужно 3-4 розетки подключенных медным проводом сечением 2.5 мм2 каждая.

Если вся техника подключается через одну единственную розетку, то сечения в 2.5 мм2 будет недостаточно, в этом случае нужно использовать провод сечением 4-6 мм2. В жилых комнатах для питания розеток можно использовать провод сечением 1.5 мм2, но окончательный выбор нужно принимать после соответствующих расчетов.

Питание всей осветительной нагрузки выполняется проводом сечением 1.5 мм2.

Необходимо понимать, что мощность на разных участках электропроводки будет разной, соответственно и сечение питающих проводов тоже разным. Наибольшее его значение будет на вводном участке квартиры, так как через него проходит вся нагрузка. Сечение вводного питающего провода выбирают 4 – 6 мм2.

При монтаже электропроводки применяют провода и кабели марки ПВС, ВВГнг, ППВ, АППВ.

Сечение кабеля по мощности (таблица)

Вот мы добрались и до сути нашей статьи. Однако всё, что было выше, упускать нельзя, а значит и мы умолчать не могли.

Если попытаться изложить мысль логично и по-простому, то через каждое условное сечение проводника может пройти ток определенной силы. Заключение это вполне логичное и теперь лишь осталось узнать эти соотношения и соотнести для разных диаметров провода, исходя из его типоряда.

Также нельзя умолчать, что здесь, при расчете сечения по току, в «игру вступает» и температура. Да, это новая составляющая – температура. Именно она способна повлиять на сечение. Как и почему, давайте разбираться.

Все мы знаем о броуновском движении. О постоянном смещении ионов в кристаллической решетке. Все это происходит во всех материалах, в том числе и в проводниках. Чем выше температура, тем больше будут эти колебания ионов внутри материала. А мы знаем, что ток — это направленное движение частиц.

Так вот, направленное движение частиц будет сталкиваться в кристаллической решетке с ионами, что приведет к повышению сопротивления для тока.

Чем выше температура, тем выше электрическое сопротивление проводника. Поэтому по умолчанию, сечение провода для определенного тока принимается при комнатной температуре, то есть при 18 градусах Цельсия. Именно при этой температуре приведены все справочные значения в таблицах, в том числе и наших.

Несмотря на то, что алюминиевые провода мы не рассматриваем в качестве проводов для электропроводки, по крайней мере, в квартире, тем не менее, они много где применяются. Скажем для проводки на улице. Именно поэтому мы также приведем значения зависимостей сечения и тока и для алюминиевых проводов.

Итак, для меди и алюминия будут следующие показатели зависимости сечения провода (кабеля) от тока (мощности). Смотрите таблицу.

Таблица проводников под допустимый максимальный ток для их использования в проводке:

С 2001 года алюминиевые провода для проводки в квартирах не применяются. (ПЭУ)

Да, здесь как заметил наш читатель, мы фактически не привели расчета, а лишь предоставили справочные данные, сведенные в таблицу, на основании этих расчетов. Но смеем вас замерить, что для расчетов необходимо перелопатить множество формул, и показателей. Начиная от температуры, удельного сопротивления, плотности тока и тому подобных.

Поэтому такие расчеты мы оставим для спецов. При этом необходимо заметить, что и они не являются окончательными, так как могут незначительно разнится, в зависимости от стандарта на материал и запаса провода по току, применяемого в разных странах.

А вот о чем мы еще хотели бы сказать, так это о переводе сечения провода в диаметр. Это необходимо, когда имеется провод, но по каким-то причинам маркировки на нем нет. В этом случае по диаметру провода можно вычислить сечения и наоборот из сечения диаметр.

Общепринятые сечения для проводки в квартире

Мы с вами много говорили о наименованиях, о материалах, об индивидуальных особенностях и даже о температуре, но упустили из вида жизненные обстоятельства.

Так если вы нанимаете электрика для того, чтобы он провел вам проводку в комнатах вашей квартиры или дома, то обычно принимаются следующие значения. Для освещения сечения провода берется в 1,5 мм 2, а для розеток в 2,5 мм 2.

Если проводка предназначена для подключения бойлеров, нагревателей, плит, то здесь уже рассчитывается сечение провода (кабеля) индивидуально.

Выбор сечения провода исходя из количества потребителей

О чем еще хотелось сказать, так это о том, что лучше использовать несколько независимых линий питания для каждого из помещений в комнате или квартире. Тем самым вы не будете применять провод с сечением 10 мм 2 для всей квартиры, проброшенный во все комнаты, от которого идут отводы.

Такой провод будет приходить на вводный автомат, а затем от него, в соответствии с мощностью потребляемой нагрузки будут разведены выбранные сечения проводов, для каждого из помещений.

Типовая принципиальная схема электропроводки для квартиры или дома с электрической плитой (с указанием сечения кабеля для электроприборов)

Токовые нагрузки в сетях с постоянным током

В сетях с постоянным током расчет сечения идет несколько по-другому. Сопротивление проводника постоянному напряжению гораздо выше, чем переменному (при переменном токе сопротивлением на длинах до 100 м вообще пренебрегают).

Кроме этого, для потребителей постоянного тока как правило очень важно, чтобы напряжение на концах было не ниже 0,5В (для потребителей переменного тока, как известно колебания напряжения в пределах 10% в любую сторону допустимы).

Есть формула, определяющая насколько упадет напряжение на концах по сравнению с базовым напряжением, в зависимости от длины проводника, его удельного сопротивления и силы тока в цепи:

U = ((p l) / S) I

    где:
  • U — напряжение постоянного тока, В
  • p — удельное сопротивление провода, Ом*мм2/м
  • l — длина провода, м
  • S — площадь поперечного сечения, мм2
  • I — сила тока, А

Зная величины указанных показателей достаточно легко рассчитать нужное Вам сечение: методом подстановки, или с помощью простейших арифметических действий над данным уравнением.

Если же падение постоянного напряжения на концах не имеет значения, то для выбора сечения можно пользоваться таблицей для переменного тока, но при этом корректировать величины тока на 15% в сторону уменьшения, т.е. при постоянном токе справочные сечения кабеля могут пропускать тока на 15 % меньше, чем указано в таблице.

Подобное правило также работает для выбора автоматических выключателей для сетей с постоянным током, например: для цепей с нагрузкой в 25А, нужно брать автомат на 15% меньшего номинала, в нашем случае подходит предыдущий типоразмер автомата — 20А.

Кабель, передающий электрический ток, – один из важнейших элементов электрической сети. В случае выхода кабеля из строя работа всей системы становится невозможной, поэтому для предотвращения отказов, а также опасности возгорания от перегрева, следует произвести точный расчёт сечения кабеля по нагрузке.

Такой расчёт дает уверенность в безопасной и надёжной работе сети и приборов, но что ещё важнее – безопасности людей.

Выбор сечения, недостаточного для токовой нагрузки, приводит к перегреву, оплавлению и повреждению изоляции, а это, в свою очередь, – к короткому замыканию и даже пожару. Так что для проведения расчётов и тщательного выбора подходящего кабеля есть масса причин.

Что необходимо для расчёта по нагрузке

Основной показатель, помогающий рассчитать сечение и марку кабеля – предельно допустимая длительная нагрузка (по току). Если проще, то это – величина тока, которую кабель способен пропускать в условиях его прокладки без перегрева достаточно долго.

Для этого необходимо простое арифметическое суммирование мощностей всех электроприборов, которые будут включаться в сеть.

Следующим важным этапом, позволяющим достичь безопасности, является расчёт сечения кабеля по нагрузке, для чего необходимо подсчитать силу тока, используя формулу:

Для однофазной сети напряжением 220 В:

    Где:
  • Р – это суммарная мощность для всех электроприборов, Вт;
  • U — напряжение сети, В;
  • COSφ — коэффициент мощности.

Для трёхфазной сети напряжением 380 В:

Наименование прибораПримерная мощность, Вт
LCD-телевизор140-300
Холодильник300-800
Пылесос800-2000
Компьютер300-800
Электрочайник1000-2000
Кондиционер1000-3000
Освещение300-1500
Микроволновая печь1500-2200

Получив точное значение величины тока, следует обратиться к таблицам, позволяющим найти кабель или провод требуемого сечения и материала. Но если полученное значение величины тока не совсем совпадает с табличным значением, то не стоит «экономить», а лучше выбрать ближайшее, но большее значение сечения кабеля.

Пример: при напряжении сети 220 В полученное значение величины тока составило 22 ампера, ближайшее большее значение (27 А) имеет медный провод или кабель из меди, сечением 2,5 мм кв. Это означает, что оптимальным выбором станет именно такой кабель, а не с сечением 1,5 мм кв., имеющим значение допустимого длительного тока 19 А.

Сечение токо-
проводящих
жил, мм
Медные жилы проводов и кабелей
Напряжение 220ВНапряжение 380В
Ток, АМощность, кВтТок, АМощность, кВт
1,5194,11610,5
2,5275,92516,5
4388,33019,8
64610,14026,4
107015,45033
168518,77549,5
2511525,39059,4
3513529,711575,9
5017538,514595,7
7021547,3180118,8
9526057,2220145,2
12030066260171,6

Если выбирается кабель с алюминиевыми жилами, то лучше взять сечение жилы не 2,5, а 4 мм кв.

Сечение токо-
проводящих
жил, мм
Алюминиевые жилы проводов и кабелей
Напряжение 220ВНапряжение 380В
Ток, АМощность, кВтТок, АМощность, кВт
2,5204,41912,5
4286,12315,1
6367,93019,8
1050113925,7
166013,25536,3
258518,77046,2
35100228556,1
5013529,711072,6
7016536,314092,4
9520044170112,2
12023050,6200132

Расчёт для помещений

Предыдущий расчёт позволил точно вычислить материал и сечение вводного кабеля, по которому будет идти общая максимальная нагрузка. Теперь следует произвести аналогичные расчёты по каждому помещению и его группам. И вот почему: нагрузка на розеточные группы может значительно отличаться.

Так, розетки с подключённой стиральной машиной и феном нагружены гораздо больше, чем розетка для миксера и кофеварки на кухне. Поэтому не стоит «упрощать» задачу, без раздумий укладывая провод сечением 2,5 квадрата на розетки, так как иногда этого просто не хватит.

Следует помнить, что суммарная нагрузка в помещении состоит из 1) силовой и 2) осветительной. И если с осветительной нагрузкой всё ясно – она выполняется медным проводом с сечением в 1,5 мм кв., то с розетками не так всё просто.

Следует помнить, что обычно кухня и ванная комната – наиболее «нагруженные» линии, так как именно там расположены холодильник, электрочайник, бойлер, микроволновка, а иногда и стиральная машинка. Поэтому лучше всего распределить эту нагрузку по различным розеточным группам, а не использовать блок на 5-6 розеток.

Иногда от «специалистов» можно услышать, что для розеток в остальных помещениях достаточно и «кабеля-полторушки», однако выдели бы вы те чёрные полосы, видные из-под обоев, которые оставляет после себя прогоревший кабель после включения в него масляного обогревателя или тепловентилятора!

    Наиболее распространенные марки проводов и кабелей:
  1. ППВ — медный плоский двух- или трехжильный с одинарной изоляцией для прокладки скрытой или неподвижной открытой проводки;
  2. АППВ — алюминиевый плоский двух- или трехжильный с одинарной изоляцией для прокладки скрытой или неподвижной открытой проводки;
  3. ПВС — медный круглый, количество жил — до пяти, с двойной изоляцией для прокладки открытой и скрытой проводки;
  4. ШВВП – медный круглый со скрученными жилами с двойной изоляцией, гибкий, для подключения бытовых приборов к источникам питания;
  5. ВВГ — кабель медный круглый, до четырех жил с двойной изоляцией для прокладки в земле;
  6. ВВП — кабель медный круглый одножильный с двойной ПВХ (поливинилхлорид) изоляцией, П — плоский (токопроводящие жилы расположены в одной плоскости).

Таблицы выбора сечения жилы при прокладке электрических проводов в резиновой или пластиковой (в том числе ПВХ=PVC) изоляции в зависимости от тока и нагрузки. Подходят для сетей 220/380В. Выбор сечения кабеля удлинителя в зависимости от длины и нагрузки.

Таблицы выбора сечения жилы при прокладке электрических проводов в резиновой или пластиковой (в том числе ПВХ=PVC) изоляции в зависимости от тока и нагрузки. Подходят для сетей 220/380В. Выбор сечения кабеля удлинителя в зависимости от длины и нагрузки.

ИТАК:

ПУЭЭ, Глава 1 нормирует допустимые длительные токи через различные виды проводов и кабелей. Другие главы регламентируют способы прокладки и прочие детали. Тем не менее мы приведем 3 таблицы для оперативного выбора площади сечения токопроводящей жилы кабеля (провода) для сетей 220/380В в зависимости от тока, нагрузки, температуры окружающей среды и способа прокладки, которыми сами пользуемся.

  • Выбираем сечения жилы (каждой) для рабочего тока или нагрузки (запоминаем ток, если прикидывали нагрузку) одиночного провода при температуре жил +65, окружающего воздуха +25 и земли + 15°С
  • Если температура не та, то смотрим поправочный коэффициент на ток в зависимости от температуры окружающей среды — если цепь вторичная = цепь управления, сигнализации, контроля, автоматики и релейной защиты электроустановок — то следующий пункт пропускаем
  • Если проводов более 1 , то смотрим поправочный коэффициент на ток в зависимости от способа прокладки
  • Делаем выбор еще раз, с учетом поправок, если нужно

Таблица 1. Выбора сечения жилы при одиночной прокладке проводов при температуре жил +65, окружающего воздуха +25 и земли + 15°С

Проложенные открыто, не пучком (в воздухе)

Проложенные в трубе

Сечение
жилы
мм2

Медь

Алюминий

Медь

Алюминий

Ток

Нагрузка, кВт

Ток

Нагрузка, кВт

Ток

Нагрузка, кВт

Ток

Нагрузка, кВт

А

1х220в

3х380в

А

1х220в

3х380в

А

1х220в

3х380в

А

1х220в

3х380в

0,5

11 2,4

0,75

15 3,3

1,0

17 3,7 6,4 14 3,0 5,3

1,5

23 5,0 8,7 15 3,3 5,7

2,0

26 5,7 9,8 21 4,6 7,9 19 4,1 7,2
14,0
3,0 5,3

2,5

30 6,6 11,0 24 5,2 9,1 21 4,6 7,9 16,0 3,5 6,0

4,0

41 9,0 15,0 32 7,0 12,0 27 5,9 10,0 21,0 4,6 7,9

6,0

50
11,0 19,0 39 8,5 14,0 34 7,4 12,0 26,0 5,7 9,8

10,0

80 17,0 30,0 60 13,0 22,0 50 11,0 19,0 38,0 8,3 14,0

16,0

100 22,0 38,0 75
16,0
28,0 80 17,0 30,0 55,0 12,0 20,0

25,0

140 30,0 53,0 105 23,0 39,0 100 22,0 38,0 65,0 14,0 24,0

35,0

170 37,0 64,0 130 28,0 49,0 135 29,0
51,0
75,0 16,0 28,0

Таблица 2. Поправочные коэффициенты на токи для кабелей, неизолированных и изолированных проводов и шин в зависимости от температуры земли и воздуха

Условная темпратура среды, °С  

Нормированная температура жил, °С  

Поправочные коэффициенты на токи при расчетной температуре среды, °С

-5 и ниже

  0

  +5

  +10

  +15

  +20

  +25

  +30

  +35

  +40

  +45

  +50

15 80 1,14 1,11 1,08 1,04 1,00 0,96 0,92 0,88 0,83 0,78 0,73 0,68
25 80 1,24 1,20 1,17 1,13 1,09 1,04 1,00 0,95 0,90
0,85
0,80 0,74
25 70 1,29 1,24 1,20 1,15 1,11 1,05 1,00 0,94 0,88 0,81 0,74 0,67
15 65 1,18 1,14 1,10 1,05 1,00 0,95 0,89 0,84 0,77 0,71 0,63 0,55
25 65 1,32 1,27 1,22 1,17 1,12 1,06 1,00 0,94 0,87 0,79 0,71 0,61
15 60 1,20 1,15 1,12 1,06 1,00 0,94 0,88 0,82 0,75 0,67 0,57 0,47
25 60 1,36 1,31 1,25 1,20 1,13 1,07 1,00 0,93 0,85 0,76 0,66 0,54
15 55 1,22 1,17 1,12 1,07 1,00 0,93 0,86 0,79 0,71 0,61 0,50 0,36
25 55 1,41 1,35 1,29 1,23 1,15 1,08 1,00 0,91 0,82 0,71 0,58 0,41
15 50 1,25 1,20 1,14 1,07 1,00 0,93 0,84 0,76 0,66 0,54 0,37
25 50 1,48 1,41 1,34 1,26 1,18 1,09 1,00 0,89 0,78 0,63 0,45

Таблица 3. Снижающие коэффициенты допустимых длительных токов в зависимости от способа прокладки (в пучках, в коробах, в лотках)

Снижающий коэффициент допустимых длительных токов для проводов, прокладываемых пучками в лотках и коробах

Снижающий коэффициент допустимых длительных токов для для проводов, прокладываемых в коробах и лотках

  • Допустимые длительные токи для проводов проложенных в коробах, а также в лотках пучками, должны приниматься как для проводов, проложенных в трубах.
  • При количестве одновременно нагруженных проводов более четырех, проложенных в трубах, коробах, а также в лотках пучками, токи для проводов должны приниматься по как для проводов, проложенных открыто (в воздухе), с введением снижающих коэффициентов
    • 0,68 для 5 и 6 проводов.
    • 0,63 для 7-9 проводов.
    • 0,6 для 10-12 проводов.
Количество проложенных проводов Снижающий коэффициент для проводов, питающих
Способ прокладки   одно жильных   много жильных отдельные электро приемники с коэффициен том использова ния до 0,7 группы электро приемников и отдельные приемники с коэф фициентом исполь зования более 0,7
Многослойно и пучками . . .  До 4 1,0
2 5-6 0,85
3-9 7-9 0,75
10-11 10-11 0,7
12-14 12-14 0,65
15-18 15-18 0,6
Однослойно 2-4 2-4 0,67
5 5 0,6
  • Допустимые длительные токи для проводов, проложенных в лотках, при однорядной прокладке (не в пучках) следует принимать, как для проводов, проложенных в воздухе.
  • Допустимые длительные токи для проводов, прокладываемых в коробах, следует принимать как для одиночных проводов, проложенных открыто (в воздухе), с применением снижающих коэффициентов, указанных в таблице.
  • При выборе снижающих коэффициентов контрольные и резервные провода и кабели не учитываются.

Расчет сечения кабеля по мощности: таблицы и формулы | Стройка/Ремонт (своими руками)

Электросети являются потенциальным источником пожарной опасности. Чтобы свести к минимуму возможность аварии, монтаж внутридомовой проводки осуществляется в строгом соответствии с установленными техническими нормативами. Рассмотрим правила правильного выбора необходимого материала, таблицу сечения кабелей по мощности, нюансы расчета нагрузки на электросети.

Для чего нужен расчёт сечения кабеля

Основное требование, предъявляемое к линиям электропередач – безопасность их эксплуатации. Поэтому, с особой внимательностью следует подходить к выбору сечения кабеля по току. Если оно окажется чересчур маленьким, проводка будет греться из-за большой нагрузки. Это, в свою очередь, способно привести к расплавлению изоляционной оплётки, короткому замыканию с последующим пожаром.

Использование проводов слишком большого сечения обезопасит дом от возгорания, но приведёт к неоправданному перерасходу денежных средств. Самый рациональный вариант при прокладке проводки – подобрать кабеля с оптимальным сечением жилы. Точные рекомендации по правильному подбору проводки даны в гл. №1.3 «Правил установки электрооборудования».

Выбор площади поперечного сечения проводника производится в соответствии со следующими параметрами:

  • Сила тока (А).
  • Мощность тока (кВт).
  • Материал изготовления проводки (медь или алюминий).
  • Количество фаз (1 или 3).

Выбираем сечение по мощности

Выбор сечения провода в зависимости от мощности тока начинается с проведения небольших расчётов. Для этого следует сложить общую мощность электрических устройств, которые будут одновременно включаться в квартире. На каждом приборе обычно указывается его мощность в ваттах или киловаттах. В будущем возможно приобретение новых бытовых электроприборов, поэтому к полученной суммарной мощности нужно прибавить ещё 1-2 киловатта.

Для устройства внутридомовой электропроводки рекомендуется использовать медные кабели. Они, хотя и стоят дороже алюминиевых, но обладают большей гибкостью, долговечностью и лучшей электропроводностью. Ниже представлены таблицы выбора сечения кабеля по мощности и силе тока для медной проводки.

Таблица 1. Вычисление мощности медной однофазной проводки напряжением в 220 вольтТаблица 2. Подбор сечения кабеля для медной трёхфазной проводки напряжением в 380 вольт.

Таблица сечения проводки в зависимости от силы и мощности тока для алюминиевых проводов выглядит иначе. В представленных выше таблицах приведены показатели соотношения сечение – ток, в зависимости от его мощности и силы.

Сила тока, проходящего по проводнику, не является постоянной величиной, и может изменяться в зависимости от следующих показателей:

  • Длина провода.
  • Размера сечения.
  • Показатель удельного сопротивления материала, из которого он сделан.
  • Температура проводника. С нагревом проводки сила тока падает.

Ниже показаны соотношения «сила тока – сечение провода» для различных вариантов прокладки. Основные цифры отдельно указаны для медных и алюминиевых проводов.

Таблица 3. Подбор сечения кабеля по мощности для алюминиевой однофазной проводки напряжением в 220 вольт.Таблица 4. Подбор сечения кабеля для алюминиевой трёхфазной проводки напряжением 380 вольт.

Как рассчитать по току

В представленных выше таблицах приведены показатели соотношения сечение – ток, в зависимости от его мощности и силы. Сила тока, проходящего по проводнику, не является постоянной величиной, и может изменяться в зависимости от следующих показателей:

  • Длина провода.
  • Размера сечения.
  • Показатель удельного сопротивления материала, из которого он сделан.
  • Температура проводника. С нагревом проводки сила тока падает.

В таблицах ниже приведены соотношения «сила тока – сечение провода» для различных вариантов прокладки. Основные цифры отдельно указаны для медных и алюминиевых проводов.

Таблица 5. Соотношение силы тока и сечение алюминиевой проводки.Таблица 6. Соотношение силы тока и сечение медной проводки.

Расчёт сечения кабеля по мощности и длине

Из-за сопротивления материала происходит некоторая потеря напряжения при прохождении тока сквозь проводник. Чем длиннее проводка, тем большая величина этих потерь. Однако, ощутимые потери могут возникнуть на линиях электропередач протяжённостью, измеряемой километрами. Для бытовой проводки они столь несущественны, что ими можно вполне пренебречь.

Рассчитываются основные показатели электротока по следующим формулам:

  • Сила тока: I = Р / (U cos ф), где:
    I — искомая сила тока.
    Р — мощность.
    U — напряжение.
    cos ф — коэффициент, применяемый для бытовой проводки. Обычно принимается за единицу.
  • Сопротивление провода: Rо=р L / S, где:
    Rо — удельное сопротивление проводника.
    р — удельное сопротивление материала, из которого он изготовлен (медь или алюминий).
    L — длина проводки.
    S — площадь сечения провода.

Открытая и закрытая прокладка проводов

При расчёте нагрузки на кабель принимается во внимание и особенности прокладки электрической линии. Существует два способа её размещения — закрытый и открытый. В стенах, изготовленных из негорючих стройматериалов – бетона, кирпича, – применяют закрытую прокладку, в специально проделанных канавках-штробах.

В деревянных зданиях проводка прокладывается открытым способом, в защитных кабель-каналах или в гофрированных трубах. Для закрытого способа монтажа используют плоские провода, а для открытой-округлые.

Источник: https://vodatyt.ru/elektrika/raschet-secheniya-kabelya.html

Вам была полезна эта статья? Ставьте палец вверх! Подпишитесь на мой канал и давайте общаться в комментариях!
С уважением, Пётр Андреевич.

Расчет сечения кабеля | СКК

При строительстве зданий и сооружений, при капитальном ремонте квартир и домов, а также при подключении какого-либо мощного электроприбора важно знать кабелем с каким сечением вести электропроводку. Если расчет сечения кабеля был произведен неправильно, равно как и не произведен вообще, возможен, по меньшей мере, выход из строя части электропроводки, а в самом худшем случае пожар, который может вызвать как огромный материальный ущерб, так и, к сожалению, человеческие жертвы.

Вот почему трудно переоценить правильный расчет сечения кабеля (провода) по мощности, по току, по напряжению, по длине и по нагрузке. Не вдаваясь в дебри, отметим, что выполняя расчет сечения кабеля по мощности нам нужно высчитать общую мощность всех потребителей и по специальным таблицам в зависимости от типа проводки и кабеля выбрать сечение. Производя расчет сечения кабеля по току, необходимо опять-таки высчитать суммарную мощность всех потребителей и разделить полученную сумму на величину напряжение сети. По полученному числу ампер при помощи специальных таблиц выбираем сечение кабеля(провода) в зависимости от типа проводки и кабеля. Выполняя расчет сечения кабеля по напряжению, следует помнить, что электрическая сеть может быть как однофазная, так и трехфазная, в соответствии, с чем вся нагрузка может концентрироваться как на одной фазе, так и делиться поровну на каждую фазу, что в свою очередь влияет на сечение жил кабеля. Рассчитывая сечение кабеля для «домашних» целей расчетами по длине можно пренебречь – расчет по длине актуален лишь для протяженных линий электропитания. Расчет кабеля по нагрузке выполняется путем сложения мощностей всех нагрузок и, согласно таблицам, в зависимости от способа прокладки проводки (скрыто или открыто) выбирается ближайшее по возрастанию значение сечения кабеля.

Расчет сечения кабеля по нагрузке

От того, насколько правильно подобрано сечение жил прокладываемых кабелей электропроводки зависит как бесперебойная работа электроприборов, так и безопасность имущества и жизни людей. Ни для кого не секрет, что в последнее время участились случаи пожаров из-за некачественной проводки. Чтобы этого избежать, необходим верный расчет сечения кабеля(провода) по нагрузке.

Как театр начинается с вешалки, так и проводка на даче, в квартире или в гараже начинается с вводного кабеля. На него выпадает самая большая нагрузка, и если по какой либо причине он не выдерживает, то велика вероятность пожара. Чтобы выяснить оптимальное сечение кабеля(провода) необходимо и достаточно прикинуть общую мощность потребления всех электроприборов на данном участке. Мощность электроприборов можно почерпнуть из паспортов приборов, из ярлыков, расположенных непосредственно на них или оценить примерно.

Так, телевизор в среднем потребляет 300 Вт, кофеварка – 1000 Вт, микроволновка 1500 Вт, электроплита 3000 Вт, стиральная машина 2200 Вт, компьютер 500 Вт, пылесос 1600 Вт, утюг – 1700 Вт и так далее. Но пользоваться приведенными здесь в достаточной мере усредненными данными следует лишь при условии отсутствия паспорта на электроприбор или ярлыка на нем. Расчет сечения кабеля(провода) по нагрузке желательно выполнять по известным конкретным данным потребляемых мощностей электроприемников.

Сложив все мощности электроприборов и освещения, у нас получится суммарная мощность потребления, даже, несмотря на то, что все приборы у нас, скорее всего, работать одновременно не будут, по крайней мере, сравнительно продолжительное время. Согласно таблицам в зависимости от способа прокладки проводки (скрыто или открыто) выбираем ближайшее по возрастанию значение сечения кабеля.

Для отходящих линий (розеточной и освещения) производим такие же вычисления. Однако желательно на розеточную группу выбирать кабель сечением минимум 2.5 мм2, а на сеть освещения — 1.5 мм2. Вот и весь расчет сечения кабеля по нагрузке.

Пример.

Суммарная мощность всех потребителей у вас получилась равной 10 кВт. Учитывая коэффициент одновременности, получим 10 000 * 0.7 = 7 кВт. Смотрим в таблицу, и видим, что 7 кВт соответствует сечение 6 мм2. Разделив мощность на напряжение, получим значение силы тока.

7 000 / 220 = 31,8 (А), то есть на вводе в квартиру, гараж или дачу необходимо поставить вводной автомат на 32 А.

Расчет сечения кабеля по длине

Электропроводка должна быть безопасна, экономична и надежна. Поэтому важен правильный расчет сечения кабеля по длине.

Если есть монтажная схема, расчет сечения кабеля(провода) по длине можно выполнить, измерив соответствующие расстояния между расположениями щитков, розеток, выключателей, распаечных коробок и так далее. Зная масштаб схемы, особого труда не составит рассчитать длины соответствующих отрезков кабеля(провода), не забывая набавлять к каждому отрезку кабеля как минимум 10 см для скруток. Если нет схемы, то длину кабеля можно оценить визуально, замерив длины линий, по которым в будущем будет проложена проводка.

Любой кабель(провод) с увеличением протяженности «теряет напряжение». Эти потери напряжения обусловлены падением напряжения в кабелях, которые соединяют электроприемник с «источником» питания. Расчет сечения кабеля по длине, учитывая потери напряжения, ведется при проектировании промышленных электрических сетей.

В «домашних» условиях, или при проектировании электропроводки небольших помещений потерями напряжения можно смело пренебречь в виду их мизерной величины. Главное в этом случае выполнить правильные расчет сечения кабеля по мощности или расчет сечения кабеля по току. А затем по специальным таблицам выбрать необходимое сечение жил кабеля.

Расчет сечения кабеля по напряжению

Расчет сечения кабеля по напряжению достаточно важен и требует внимания. Осуществляя расчет сечения кабеля по напряжению, следует иметь в виду, что электрическая сеть может быть как однофазная (рабочее напряжение 220 В), так и трехфазная (3*220 / 380 В). То есть потребляемая мощность может приходить к дому или крупному приемнику электроэнергии как однофазной нагрузкой, так и трехфазной.

Например, суммарная потребляемая мощность гаража у нас, к примеру, 20 кВт. В однофазной проводке на фазу будет идти вся нагрузка 20 кВт, а в трехфазной проводке — лишь 6.6 кВт. Соответственно, при большей нагрузке на жилу нам будут необходимы большие сечения кабеля(провода), при меньших нагрузках – соответственно меньшие. Единственный момент: для однофазной проводки нам понадобится трехжильный кабель, а для трехфазной проводки – пятижильный. Поэтому уменьшение сечения кабеля одновременно увеличивает количество жил.

Также выполняя расчет сечения кабеля(провода) по напряжению, стоит помнить, что некоторые электроприборы и двигатели работают только от сети 380 В.

Расчет сечения кабеля по току

Для качественной прокладки электропроводки, чтобы избежать ненужных неприятностей и бед, да и просто, чтобы спать спокойно, жизненно необходимо внимательно выполнить расчет сечения кабеля по току. Чтобы выполнить расчет сечения кабеля по току вам потребуется высчитать ток, который будет проходить по нашей проводке. Номинальный ток высчитывается при помощи суммарной мощности нагрузки. Суммарная мощность нагрузки высчитывается соответственно сложением мощностей всех электроприборов, которые будут брать электроэнергию с нашей линии. Нужно учитывать все мощности, находящиеся на искомом участке.

Например, у нас на участке 3 светильника по 100 Вт, холодильник Атлант 200 Вт, микроволновка Samsung 1100 Вт, электрочайник Bosch 2200 Вт. Проводка у нас будет однофазная и будет проложена скрыто. Суммарная мощность у нас будет равна P=100*3+200+1100+2200=3800 Вт.

От суммарной мощности находим искомый ток по формуле, знакомой еще со школы:

I = P/U*cos?,

где P – наша суммарная мощность, I – номинальный ток, U – напряжение, cos? — коэффициент мощности. Сos? в нашем случае практически равен 1, соответственно им можно пренебречь.

Согласно формуле, I = 3800/220*1 = 17.3 А. Смотрим по таблице кабель, способный в скрытой проводке длительно держать 17.3 А – это медный кабель с минимальным сечением 2 мм2. Для запаса, используем для проводки медный кабель, с сечением 3*2.5 мм2. Расчет сечения кабеля по току завершен.

Расчет сечения кабеля по мощности

Представим, что нам, например, нужно выбрать кабель для электропроводки квартиры. В квартире мы имеем однофазную проводку, с рабочим напряжением 220 В. Чтобы подобрать необходимый кабель нам необходим расчет сечения кабеля по мощности. Чтобы это осуществить, нужно всего лишь посчитать суммарную мощность возможных потребителей электрической энергии. На всех электроприборах, как правило, присутствует ярлык завода-изготовителя о мощности потребления. Кроме электроприборов необходимо просуммировать мощность всех осветительных приборов. Допустим, в результате сложения мощности всех утюгов, холодильников, телевизоров, микроволновок, стиральных машин, чайников и остальных электроприборов вместе с освещением у вас получилось 7кВт. Получается, нам необходимо сделать расчет сечения кабеля(провода) по мощности 7 кВт. Хотя все электроприборы одновременно обычно не включаются, будем считать по максимуму. Для больших промышленных объектов для точного вычисления нагрузки используются коэффициенты одновременности, спроса и так далее, однако в наших «домашних» условиях обойдемся без этих сложностей.

Тем самым осуществим расчет сечения кабеля по мощности 7 кВт. Согласно таблицам ПУЭ выясним, что такую мощность выдержит медный кабель 3х6 или алюминиевый кабель 3х10. Помня, что скупой платит дважды, не экономьте на сечении кабеля!

Расчет сечения кабеля по мощности: примеры, формулы и таблицы

Для того чтобы электропроводка функционировала безупречно, важно правильно подобрать сечение проводов и сделать грамотный расчет мощности, ведь от этих показателей зависят другие характеристики. Ток движется по проводам точно так же, как вода поступает по трубам.

Кабель для электропроводки

От качества проведённых электромонтажных работ зависит безопасность всего помещения. Здесь особенно важно правильно выбрать такой параметр, как сечение кабеля. Для того чтобы провести расчёт сечения кабеля по мощности, необходимо знать технические характеристики всех потребителей электроэнергии, которые будут к нему подключены. Также следует учесть длину проводки и то, каким способом она будет установлена.

Ток движется по проводам подобно тому, как вода течёт по трубе. Как в водопроводную трубу нельзя поместить жидкость большего объёма, так и по кабелю невозможно пропустить больше определённого количества тока. Кроме того, стоимость кабеля напрямую зависит от его сечения. Чем больше сечение, тем цена кабеля будет выше.

Водопроводная труба по сечению большая, чем нужно, стоит дороже, а слишком узкая не пропустит нужного количества воды. То же самое происходит и с током с той лишь разницей, что выбор кабеля, по сечению меньше заданного значения, намного опаснее. Такой провод все время перегревается, мощность тока в нем повышается. Из-за этого свет в помещении будет произвольно вырубаться, а в худшем случае произойдёт короткое замыкание, начнётся пожар.

Кабель в разрезе

В том, что выбранное сечение кабеля будет больше необходимого, ничего страшного нет. Наоборот, проводка, где мощность и сечение превышают нужное значение, прослужит намного дольше, но стоимость всех электромонтажных работ сразу вырастет как минимум в 2-3 раза, ведь основные затраты на электроснабжение заключаются именно в стоимости проводов.
Правильно выбранное сечение позволит:

  • избежать перегрева проводов;
  • не допустить короткого замыкания;
  • сэкономить на стоимости ремонта.

Вычисление с помощью формул

Достаточная площадь сечения даст возможность проходить максимальному току по проводам без перегрева. Поэтому при проектировании электропроводки, прежде всего, находят оптимальное сечение провода в зависимости от потребляемой мощности. Для вычисления этого значения следует подсчитать общий ток. Его определяют, исходя из мощности всех приборов, подключённых к кабелю.

Чтобы выбрать оптимальное сечение провода, зная мощность, следует вспомнить закон Ома, а также правила электродинамики и другие электромеханические формулы. Так, силу тока (I) для участка сети с напряжением 220 Вольт, а именно такое напряжение используется для домашней сети, рассчитывают формулой:

I=(P1+P2+…+Pn)/220, где:

(P1+P2+…+Pn) – суммарная мощность каждого используемого электроприбора.

Для сетей с напряжением 380 Вольт:

I=(P1+P2+…+Pn)/ √3/380.

Показатели мощности некоторых бытовых электроприборов

ЭлектроприборМощность, ВтЭлектроприборМощность, Вт
Блендердо 500Полотенцесушитель900-1700
Вентилятор750-1700Посудомоечная машина2000
Видеомагнитофондо 500Пылесос400-2000
Водонагреватель накопительный1200-1500Соковыжималкадо 1000
Водонагреватель проточный2000-5000Стиральная машина3000
Вытяжка (вентиляция)500-1000Стиральная машина с сушилкой3500
Гриль1200-2000Сушка для рук800
Духовка1000-2000Телевизор100-400
Компьютер400-750Тостер600-1500
Кондиционер1000-3000Увлажнитель воздуха200
Кофеварка800-1500Утюг500-2000
Кухонный комбайндо 100Фен для волос450-2000
Микроволновая печь850Фритюрница1500
Микроволновая печь комбинированная2650Холодильник200-600
Миксердо 500Электробритвадо 100
Мясорубка500-1000Электролампы20-250
Обогреватель1000-2400Электроплита8000-10000
Пароварка500-1000Электрочайник1000-2000

Но это расплывчатые формулы и упрощённый расчет. В подробных вычислениях учитывается величина допустимых нагрузок, которая для медного кабеля составит 10А/мм², а для алюминиевого – 8 А/мм². Нагрузка определяет, какой по силе ток может проходить через единицу площади беспрепятственно.

Коррекция показателей мощности

Также при расчете добавляют поправку в виде коэффициента спроса (Kс). Этот коэффициент показывает, какие приборы используются в сети постоянно, а какие в течение определенного времени. Специальный калькулятор и таблицы, где показан расчет мощности, упрощают все эти вычисления.

Коэффициенты спроса приемников собственных нужд (Kс)

Наименование приемникаКоэффициент спроса
Освещение ОРУ:
при одном0,5
при нескольких0,35
Освещение помещений0,6-0,7
Охлаждение трансформаторов0,8-0,85
Компрессоры0,4
Зарядно-подзарядные устройства0,12
Электроподогрев выключателей и электроотопление1,0

Но как поступить, если в характеристиках указаны 2 вида мощности: активная и реактивная? Причём первая из них измеряет в привычных всем кВ, а вторая – кВА. В наших сетях течёт переменный ток, величина которого меняется во времени. Поэтому для всех потребителей существует активная мощность, которая рассчитывается как среднее значение всех мгновенных переменных тока и мощности. К приборам с активной мощностью относятся лампы накаливания, электронагреватели. У таких потребителей энергии фазы тока и напряжения совпадают. Если же в электрической цепи задействованы агрегаты, накапливающие энергию, например, трансформаторы или электродвигатели, то у них возможны отклонения по амплитуде. За счёт этого явления и возникает реактивная мощность.

Для сетей, где существует реактивная и активная мощность, надо взять в расчет ещё одну поправку – коэффициент мощности (cosφ) или реактивную составляющую.

Таким образом, получается формула:

S= Kс*(P1+P2+…+Pn)/(220*cosφ*Рд), где:

  • S – площадь сечения,
  • Рд – допустимая величина нагрузки.

Помимо этого, рассматриваются возможные потери мощности по току, которые возникают во время прохождения по проводам. При использовании кабеля с несколькими жилами нужно умножить величину потерь на число этих жил.

Важно! Для всех этих вычислений потребуется не просто калькулятор, но и глубокие познания в области физики. Сделать точный расчет сразу без теоретических знаний не получится.

Поиск площади по диаметру

Иногда даже скрупулезный расчет не помогает, в цепи происходит короткое замыкание. Это связано с тем, что заявленные технические характеристики часто не соответствуют реальному значению. Поэтому, чтобы узнать, как сделать расчет мощности, важно быть уверенным в том, что в магазине предложат подходящий электропровод по сечению. Для этого воспользуемся простой формулой:

S=0,785d2, где:

  • d – это диаметр жилы;
  • S – площадь сечения.

Определить точный диаметр провода, сделать расчет сечения можно с помощью штангенциркуля или микрометра, что точнее.

Микрометр

Если кабель состоит из нескольких тонких проводов, то сначала смотрят диаметр одного из них, а потом полученные данные умножают на их количество:

S

Как правильно выбрать сечение кабеля, таблицы сечения по мощности и току

Выбирая кабель особенно важно подобрать правильное сечение для надёжной и безаварийной работы электрооборудования. Для этого используются специальные таблицы выбора сечения кабеля, учитывающие металл, из которого изготовлена токопроводящая жила, материал изоляции и другие параметры.
 

Таблица сечения кабеля по мощности и току

Обычно для практических нужд достаточно использовать таблицу сечения кабеля, которая находится в Правилах Устройства Электроустановок в таблицах 1.3.4 и 1.3.5.

Также можно использовать следующие таблицы.

Для гибкого шнура и кабеля с медной жилой (ПВС, ШВВП, КГ)

Для силового кабеля с медной жилой (ВВГ)

 


Для силового кабеля с алюминиевой жилой (АВВГ)

 

В этих таблицах указаны необходимые сечения алюминиевых и медных кабелей для различных токовых нагрузок и условий прокладки. Тип изоляции — резиновая и виниловая, аналогичен большинству видов изоляционных материалов.

Выбор производится по номинальному току нагрузки. Если ток неизвестен, то он вычисляется исходя из мощности устройства, количества фаз и напряжения сети.
 

Какие параметры необходимо учесть для выбора правильного сечения кабеля

Для надёжной работы электроприборов при выборе кабеля по сечению учитываются различные факторы, основными из которых являются следующие:

  • номинальный ток нагрузки;
  • материал токопроводящей жилы;
  • тип изоляции;
  • способ прокладки;
  • длина кабеля.

Перед тем, как рассчитать сечение кабеля, необходимо определить эти параметры.
 

Способы расчёта сечения кабелей

Есть два способа определения необходимого сечения кабеля. При расчёте необходимо применять оба метода и использовать большую из полученных величин.

Расчёт сечения по нагреву

Во время протекания электрического тока по кабелю он греется. Допустимая температура нагрева и сечение провода зависят от типа изоляции и способов прокладки. При недостаточном сечении токопроводящей жилы она нагревается до недопустимой температуры, что может привести к разрушению изоляции, короткому замыканию и пожару.

Совет! Для тщательного расчёта необходимо использовать специальные таблицы, программы или онлайн-калькуляторы, но для большинства практических задач допускается применить таблицу, которую можно найти в ПУЭ, п. 1.3.10.

Расчёт сечения по допустимым потерям напряжения

Токопроводящая жила в проводе обладает сопротивлением и при прохождении по ней тока, согласно закону Ома, происходит падение напряжения. Величина этого падения растёт при уменьшении сечения кабеля и увеличении его длины.

При прокладке кабеля большой длины его сечение, необходимое для уменьшения потерь, может многократно превышать величину, выбранную по допустимому нагреву. Для расчёта используются специальные формулы, программы и онлайн-калькуляторы.

Совет! При подключении устройств, работающих на пониженном напряжении, блок питания располагается как можно ближе к аппарату.
 

Расчёт сечения для однофазной и трехфазной сети

Выбор кабеля производится по току нагрузки, но если он неизвестен, то выполняется выбор сечения кабеля по мощности. Методы расчёта различные для однофазных и трёхфазных нагрузок.

Расчёт тока однофазных нагрузок

Для вычисления этого параметра необходимо разделить мощность устройства на напряжение сети

I=P/U

В однофазной сети ~220В допускается использование упрощённой формулы

I=4,5P

Расчёт токов в трёхфазной сети

В трёхфазной сети 380В есть два вида нагрузок, ток которых вычисляется по-разному:

  • Электродвигатели. Для расчёта необходимо учесть КПД и cosφ, но допускается использование формулы

I=2P

  • Нагреватели. Эти установки рассматриваются как три однофазных нагревателя, и применяется формула

I=(P/3)/U=4,5(P/3)

Важно! При подключении электроплиты, расчёт производится по самому мощному нагревателю или двум, в зависимости от схемы аппарата.
 

Какое сечения кабеля выбрать в квартиру или частный дом

При проектировании электропроводки в квартире или частном доме используются гибкие медные провода ПВС или ШВВП. В этом случае допускается не производить расчёт проводов, а использовать стандартные сечения токопроводящих жил:

  1. Освещение. Общие провода 1,5мм², подключение отдельных светильников 0,5-1мм².
  2. Комнатные розетки, кондиционеры и мелкая кухонная техника. Общий кабель 2,5мм², опуск к отдельным розеткам 1,5мм².
  3. Посудомоечные и стиральные машины, электродуховки, бойлеры. Это установки повышенной мощности и розетка для каждого из этих устройств подключается отдельным кабелем 1,5мм². При установке двух таких устройств рядом возле розеток монтируется переходная коробка с клеммником, который подключается кабелем 2,5мм². При установке нескольких мощных аппаратов сечение общего провода выбирается по суммарному току этих установок.
  4. Нагреватели проточной воды. Устройство для кухни мощностью 3кВт присоединяется проводом 1,5мм², для ванной мощностью 5кВт кабелем 2,5мм², идущим прямо из вводного щитка.
  5. Электроплита. Двухконфорочная плита подключается кабелем 2,5мм², четырёхконфорочная в однофазной сети присоединяется проводом 4мм². В трёхфазной достаточно сечения 2,5мм².
  6. Электроотопление. Сечение общего кабеля определяется мощностью системы. При значительно количестве нагревателей и большой протяжённости кабеля допускается установка последовательно нескольких кабелей разного сечения. При наличии в доме трёхфазной электропроводки целесообразно электроконвектора и тёплые полы в разных комнатах подключить к различным фазам. Это позволит уменьшить сечение питающих кабелей.

Знание того, как правильно рассчитать сечение кабеля, поможет выполнить монтаж электропроводки без привлечения проектных организаций.

Калькулятор расчета сечения силового провода – Ученик – общение музыкой

Проводом какого сечения нужно подключать те или иные устройства в бортовую сеть автомобиля? Как сечение провода влияет на падение напряжения на нагрузке?

Чтобы рассчитать это и был создан этот калькулятор. Он позволяет рассчитать необходимое сечение провода в зависимости от материала из которого изготовлены провода, напряжения бортовой сети, мощности нагрузки, длины проводов и допустимого (по Вашему мнению) падения напряжения в проводах.

 

 

 

Для простоты расчетов сечения провода приводим следующую таблицу перевода AWG (American Wire Gauge – обозначения сечения провода по американскому стандарту) в метрические характеристики провода. Сила максимального тока, указанная в правом столбце, дана для долговременной нагрузки с запасом по возможности увеличения плотности тока до 25-50%. Однако, результатом такого увеличения плотности тока будет большее падение напряжения на подключенном потребителе.

 

Номер
AWG
 
Диаметр,
мм 
Площадь
сечения, кв.мм 
Maкс. ток, при
5 А/кв.мм 
000011.70107.459537.3
00010.4084.906424.5
009.3067.895339. 5
08.3054.079270.4
17.3542.385211.9
26.5433.617168.1
35.8326.654133.3
45.1921.137105.7
54.6216.76383.8
64.1213.29366. 5
73.6710.54452.7
83.268.36341.8
92.916.62933.1
102.595.25826.3
112.314.17120.9
122.053.30916.5
131.832.62313. 1
141.632.08110.4
151.451.6508.3
161.291.3086.5
171.151.0385.2
181.020.8234.1
190.910.6533.3
200.810.5172.6
210. 720.4102.1
220.640.3261.6
230.570.2581.3
240.510.2051.0
250.460.1630.8
260.410.1290.6
270.360.1020.5
280. 320.0810.4
290.290.0640.3
300.260.05100.3
310.230.0400.2
320.200.0320.2
330.180.0250.1
340.160.0200.1
350.140. 0160.1
360.130.0130.1
370.110.0100.1
380.100.0080.0

 

Учебное пособие по физике: электрическое сопротивление

Электрон, движущийся по проводам и нагрузкам внешней цепи, встречает сопротивление. Сопротивление препятствует прохождению заряда. Для электрона путешествие от терминала к терминалу не является прямым маршрутом. Скорее, это зигзагообразный путь, который возникает в результате бесчисленных столкновений с неподвижными атомами в проводящем материале. Электроны сталкиваются с сопротивлением — препятствием для их движения. В то время как разность электрических потенциалов, установленная между двумя выводами , способствует перемещению заряда , а препятствует его движению.Скорость, с которой заряд проходит от терминала к терминалу, является результатом совместного действия этих двух величин.

Переменные, влияющие на электрическое сопротивление

Поток заряда по проводам часто сравнивают с потоком воды по трубам. Сопротивление потоку заряда в электрической цепи аналогично эффектам трения между водой и поверхностями трубы, а также сопротивлению, создаваемому препятствиями на пути.Именно это сопротивление препятствует потоку воды и снижает как скорость потока, так и скорость сноса . Подобно сопротивлению потоку воды, общее сопротивление потоку заряда в проводе электрической цепи зависит от некоторых четко идентифицируемых переменных.

Во-первых, общая длина проводов влияет на величину сопротивления. Чем длиннее провод, тем большее сопротивление будет. Существует прямая зависимость между величиной сопротивления, с которым сталкивается заряд, и длиной провода, который он должен пройти.В конце концов, если сопротивление возникает в результате столкновений между носителями заряда и атомами провода, то в более длинном проводе, вероятно, будет больше столкновений. Больше столкновений означает большее сопротивление.

Во-вторых, на величину сопротивления влияет площадь поперечного сечения проводов. Более широкие провода имеют большую площадь поперечного сечения. Вода будет течь по более широкой трубе с большей скоростью, чем по узкой. Это можно объяснить меньшим сопротивлением, которое присутствует в более широкой трубе.Таким же образом, чем шире провод, тем меньше будет сопротивление прохождению электрического заряда. Когда все другие переменные одинаковы, заряд будет течь с большей скоростью через более широкие провода с большей площадью поперечного сечения, чем через более тонкие провода.

Третья переменная, которая, как известно, влияет на сопротивление потоку заряда, — это материал, из которого сделан провод. Не все материалы созданы равными с точки зрения их проводящей способности. Некоторые материалы являются лучшими проводниками, чем другие, и обладают меньшим сопротивлением потоку заряда.Серебро — один из лучших проводников, но никогда не используется в проводах бытовых цепей из-за своей стоимости. Медь и алюминий являются одними из наименее дорогих материалов с подходящей проводящей способностью, позволяющей использовать их в проводах бытовых цепей. На проводящую способность материала часто указывает его удельное сопротивление . Удельное сопротивление материала зависит от электронной структуры материала и его температуры. Для большинства (но не для всех) материалов удельное сопротивление увеличивается с повышением температуры.В таблице ниже приведены значения удельного сопротивления для различных материалов при температуре 20 градусов Цельсия.

Материал

Удельное сопротивление (Ом • метр)

Серебро

1,59 х 10 -8

Медь

1. 7 х 10 -8

Золото

2,2 х 10 -8

Алюминий

2,8 х 10 -8

Вольфрам

5,6 х 10 -8

Утюг

10 х 10 -8

Платина

11 х 10 -8

Свинец

22 х 10 -8

Нихром

150 х 10 -8

Углерод

3.5 х 10 -5

Полистирол

10 7 — 10 11

Полиэтилен

10 8 — 10 9

Стекло

10 10 — 10 14

Твердая резина

10 13

Как видно из таблицы, существует широкий диапазон значений удельного сопротивления для различных материалов. Материалы с более низким сопротивлением обладают меньшим сопротивлением потоку заряда; они лучше дирижеры. Материалы, показанные в последних четырех строках приведенной выше таблицы, обладают таким высоким удельным сопротивлением, что их даже нельзя рассматривать как проводники.

Посмотри! Используйте виджет Resistivity of a Material , чтобы найти удельное сопротивление данного материала. Введите название материала и нажмите кнопку Submit , чтобы узнать его удельное сопротивление.

Математическая природа сопротивления

Сопротивление — это числовая величина, которую можно измерить и выразить математически. Стандартной метрической единицей измерения сопротивления является ом, представленный греческой буквой омега -. Электрическое устройство с сопротивлением 5 Ом будет представлено как R = 5 . Уравнение, представляющее зависимость сопротивления ( R ) проводника цилиндрической формы (например,, провод) от влияющих на него переменных равно

, где L представляет длину провода (в метрах), A представляет площадь поперечного сечения провода (в метрах 2 ) и представляет удельное сопротивление материала (в Ом • метр). В соответствии с вышеизложенным, это уравнение показывает, что сопротивление провода прямо пропорционально длине провода и обратно пропорционально площади поперечного сечения провода.Как показывает уравнение, знание длины, площади поперечного сечения и материала, из которого сделан провод (и, следовательно, его удельного сопротивления), позволяет определить сопротивление провода.

Расследовать!

Резисторы — один из наиболее распространенных компонентов электрических цепей. На большинстве резисторов нанесены цветные полосы или полосы. Цвета отображают информацию о значении сопротивления.Возможно, вы работаете в лаборатории и вам нужно знать сопротивление резистора, используемого в лаборатории. Используйте виджет ниже, чтобы определить значение сопротивления по цветным полосам.

Проверьте свое понимание

1. В бытовых цепях часто используются провода двух разной ширины: калибра 12 и калибра 14. Проволока 12-го калибра имеет диаметр 1/12 дюйма, а проволока 14-го калибра — 1/14 дюйма.Таким образом, провод 12-го калибра имеет более широкое сечение, чем провод 14-го калибра. Цепь на 20 А, используемая для настенных розеток, должна быть подключена с использованием провода 12-го калибра, а цепь на 15 А, используемая для цепей освещения и вентиляторов, должна быть подключена с помощью провода 14-го калибра. Объясните физику такого электрического кода.


2. Основываясь на информации, указанной в предыдущем вопросе, объясните риск, связанный с использованием провода 14-го калибра в цепи, которая будет использоваться для питания 16-амперной пилы.


3. Определите сопротивление медного провода 12 калибра длиной 1 милю. Дано: 1 миля = 1609 метров и диаметр = 0,2117 см.


4. Две проволоки — A и B — круглого сечения имеют одинаковую длину и изготовлены из одного материала. Тем не менее, сопротивление провода A в четыре раза больше, чем у провода B.Во сколько раз диаметр проволоки B больше диаметра проволоки A?

15.3 Сопротивление и удельное сопротивление — Физика Дугласского колледжа 1104 Пользовательский учебник — Зима и лето 2020

Задачи и упражнения

1: Каково сопротивление отрезка медного провода 12 калибра длиной 20,0 м и диаметром 2,053 мм?

2: Диаметр медной проволоки нулевого сечения равен 8.252 мм. Найдите сопротивление такого провода длиной 1,00 км, используемого для передачи энергии.

3: Если вольфрамовая нить диаметром 0,100 мм в лампочке должна иметь сопротивление [латекс] \ boldsymbol {0.200 \; \ Omega} [/ latex] при 20,0 ° C, какой длины она должна быть?

4: Найдите отношение диаметра алюминиевого провода к медному, если они имеют одинаковое сопротивление на единицу длины (как в бытовой электропроводке).

5: Какой ток протекает через 2.3 \; \ textbf {V}} [/ latex] применяется к нему? (Такой стержень можно использовать, например, для изготовления детекторов ядерных частиц.)

6: (a) До какой температуры вы должны нагреть медный провод, изначально равный 20,0 ° C, чтобы удвоить его сопротивление, не обращая внимания на любые изменения размеров? (б) Происходит ли это в бытовой электропроводке при обычных обстоятельствах?

7: Резистор из нихромовой проволоки используется в приложениях, где его сопротивление не может измениться более чем на 1,00% от значения 20.0ºC. В каком температурном диапазоне его можно использовать?

8: Из какого материала изготовлен резистор, если его сопротивление на 40,0% больше при 100 ° C, чем при 20,0 ° C?

9: Электронное устройство, предназначенное для работы при любой температуре в диапазоне от –10,0 ° C до 55,0 ° C, содержит резисторы из чистого углерода. В какой степени их сопротивление увеличивается в этом диапазоне?

10: (a) Из какого материала сделана проволока, если она имеет длину 25,0 м, диаметр 0,100 мм и сопротивление [латекс] \ boldsymbol {77.7 \; \ Omega} [/ latex] при 20,0ºC? (б) Каково его сопротивление при 150 ° C?

11: Если принять постоянный температурный коэффициент удельного сопротивления, каков максимальный процент уменьшения сопротивления константановой проволоки, начиная с 20,0 ° C?

12: Проволока протягивается через матрицу, растягивая ее в четыре раза по сравнению с исходной длиной. В какой степени увеличивается его сопротивление?

13: Медный провод имеет сопротивление [латекс] \ boldsymbol {0,500 \; \ Omega} [/ латекс] при 20.{\ circ} \ textbf {C}} [/ latex]), когда он находится при той же температуре, что и пациент. Какова температура пациента, если сопротивление термистора при этой температуре составляет 82,0% от его значения при 37,0 ° C (нормальная температура тела)? (b) Отрицательное значение [латекс] \ boldsymbol {\ alpha} [/ latex] не может поддерживаться при очень низких температурах. Обсудите, почему и так ли здесь. (Подсказка: сопротивление не может стать отрицательным.)

15: интегрированные концепции

(a) Повторите упражнение 2 с учетом теплового расширения вольфрамовой нити.{\ circ} \ textbf {C}} [/ латекс]. б) На какой процент ваш ответ отличается от приведенного в примере?

16: Необоснованные результаты

(a) До какой температуры нужно нагреть резистор из константана, чтобы удвоить его сопротивление, при условии постоянного температурного коэффициента удельного сопротивления? б) разрезать пополам? (c) Что необоснованного в этих результатах? (d) Какие предположения необоснованны или какие посылки несовместимы?

Resistivity and Resistance — University Physics Volume 2

Теперь рассмотрим сопротивление провода или компонента.Сопротивление — это мера того, насколько сложно провести ток через провод или компонент. Сопротивление зависит от удельного сопротивления. Удельное сопротивление — это характеристика материала, используемого для изготовления провода или другого электрического компонента, тогда как сопротивление — это характеристика провода или компонента.

Чтобы рассчитать сопротивление, рассмотрите сечение проводящего провода с площадью поперечного сечения A , длиной L и удельным сопротивлением. Батарея подключается к проводнику, обеспечивая разность потенциалов на нем ((рисунок)).Разность потенциалов создает электрическое поле, пропорциональное плотности тока, согласно.

Величина электрического поля на сегменте проводника равна напряжению, деленному на длину,, а величина плотности тока равна току, деленному на площадь поперечного сечения. Используя эту информацию и вспомнив что электрическое поле пропорционально удельному сопротивлению и плотности тока, мы можем видеть, что напряжение пропорционально току:

Единицей измерения сопротивления является ом,.Для данного напряжения, чем выше сопротивление, тем ниже ток.

Резисторы

Обычным компонентом электронных схем является резистор. Резистор можно использовать для уменьшения протекания тока или обеспечения падения напряжения. (Рисунок) показывает символы, используемые для резистора в принципиальных схемах цепи. Два широко используемых стандарта для принципиальных схем предоставлены Американским национальным институтом стандартов (ANSI, произносится как «AN-см.») И Международной электротехнической комиссией (IEC).Обе системы обычно используются. Мы используем стандарт ANSI в этом тексте для его визуального распознавания, но отметим, что для более крупных и сложных схем стандарт IEC может иметь более четкое представление, что упрощает чтение.

Обозначения резистора, используемого в принципиальных схемах. (а) символ ANSI; (b) символ IEC.

Зависимость сопротивления материала и формы от формы

Резистор можно смоделировать как цилиндр с площадью поперечного сечения A и длиной L , сделанный из материала с удельным сопротивлением ((рисунок)).Сопротивление резистора составляет.

Модель резистора в виде однородного цилиндра длиной L и площадью поперечного сечения A . Его сопротивление потоку тока аналогично сопротивлению, оказываемому трубой потоку жидкости. Чем длиннее цилиндр, тем больше его сопротивление. Чем больше его площадь поперечного сечения A , тем меньше его сопротивление.

Наиболее распространенным материалом для изготовления резистора является углерод. Углеродная дорожка обернута вокруг керамического сердечника, к нему прикреплены два медных вывода.Второй тип резистора — это металлопленочный резистор, который также имеет керамический сердечник. Дорожка сделана из материала оксида металла, который имеет полупроводниковые свойства, аналогичные углеродным. Опять же, в концы резистора вставляются медные провода. Затем резистор окрашивается и маркируется для идентификации. Резистор имеет четыре цветные полосы, как показано на (Рисунок).

Многие резисторы напоминают рисунок, показанный выше. Четыре полосы используются для идентификации резистора. Первые две цветные полосы представляют собой первые две цифры сопротивления резистора. {5} \ phantom {\ rule {0.2em} {0ex}} \ text {Ω} Ошибка пакета inputenc: символ Юникода ± (U + 00B1) начальный текст: … ext {Ω} \ phantom {\ rule {0.2em} {0ex}} \ text {±} Файл завершился при сканировании использования \ text @. Экстренная остановка.

.

Сопротивление может быть разным. Некоторые керамические изоляторы, например те, которые используются для поддержки линий электропередач, имеют сопротивление или более. Сухой человек может иметь сопротивление руки к ноге, тогда как сопротивление человеческого сердца составляет около.Кусок медного провода большого диаметра длиной в метр может иметь сопротивление, а сверхпроводники вообще не имеют сопротивления при низких температурах. Как мы видели, сопротивление связано с формой объекта и материалом, из которого он состоит.

Плотность тока, сопротивление и электрическое поле для токоведущего провода. Рассчитайте плотность тока, сопротивление и электрическое поле медного провода длиной 5 м и диаметром 2,053 мм (калибр 12), по которому течет ток 0,5 м.

Стратегия

Мы можем рассчитать плотность тока, сначала найдя площадь поперечного сечения провода, которая есть, и определение плотности тока. Сопротивление можно найти, используя длину провода, площадь и удельное сопротивление меди, где. Удельное сопротивление и плотность тока можно использовать для определения электрического поля.

Решение Сначала мы вычисляем плотность тока:

Сопротивление провода

Наконец, мы можем найти электрическое поле:

Значимость Исходя из этих результатов, неудивительно, что медь используется для проводов, проводящих ток, потому что сопротивление довольно мало.Обратите внимание, что плотность тока и электрическое поле не зависят от длины провода, но напряжение зависит от длины.

Сопротивление объекта также зависит от температуры, поскольку оно прямо пропорционально. Мы знаем, что для цилиндра L и A не сильно изменяются с температурой, R имеет ту же температурную зависимость, что и (Исследование коэффициентов линейного расширения показывает, что они примерно на два порядка меньше, чем типичные температурные коэффициенты удельного сопротивления, поэтому влияние температуры на L и A примерно на два порядка меньше, чем на

— это температурная зависимость сопротивления объекта, где — исходное сопротивление (обычно принимается равным R — сопротивление после изменения температуры. Цветовой код показывает сопротивление резистора при температуре.

Многие термометры основаны на влиянии температуры на сопротивление ((Рисунок)). Один из наиболее распространенных термометров основан на термисторе, полупроводниковом кристалле с сильной температурной зависимостью, сопротивление которого измеряется для определения его температуры. Устройство небольшое, поэтому быстро приходит в тепловое равновесие с той частью человека, к которой прикасается.

Эти знакомые термометры основаны на автоматическом измерении сопротивления термистора в зависимости от температуры.

Проверьте свои знания Тензодатчик — это электрическое устройство для измерения деформации, как показано ниже. Он состоит из гибкой изолирующей основы, поддерживающей рисунок из проводящей фольги. Сопротивление фольги изменяется по мере растяжения основы. Как меняется сопротивление тензодатчика? Влияет ли тензодатчик на изменение температуры?

Рисунок фольги растягивается при растяжении основы, а дорожки фольги становятся длиннее и тоньше. Поскольку сопротивление рассчитывается как, сопротивление возрастает по мере того, как дорожки из фольги растягиваются. При изменении температуры меняется и удельное сопротивление фольги, меняя сопротивление. Один из способов борьбы с этим — использовать два тензодатчика, один используется в качестве эталона, а другой — для измерения деформации. Два тензодатчика поддерживают при постоянной температуре

Сопротивление коаксиального кабеля Длинные кабели иногда могут действовать как антенны, улавливая электронные шумы, которые являются сигналами от другого оборудования и приборов.Коаксиальные кабели используются во многих случаях, когда требуется устранение этого шума. Например, их можно найти дома через кабельное телевидение или другие аудиовизуальные соединения. Коаксиальные кабели состоят из внутреннего проводника радиуса, окруженного вторым внешним концентрическим проводником радиуса ((Рисунок)). Пространство между ними обычно заполнено изолятором, например полиэтиленовым пластиком. Между двумя проводниками возникает небольшой ток радиальной утечки. Определите сопротивление коаксиального кабеля длиной L .

Коаксиальные кабели состоят из двух концентрических жил, разделенных изоляцией. Они часто используются в кабельном телевидении или других аудиовизуальных средствах связи.

Стратегия Мы не можем использовать уравнение напрямую. Вместо этого мы смотрим на концентрические цилиндрические оболочки толщиной dr и интегрируем.

Решение Сначала находим выражение для dR , а затем интегрируем от до,

Значение Сопротивление коаксиального кабеля зависит от его длины, внутреннего и внешнего радиусов, а также удельного сопротивления материала, разделяющего два проводника.Поскольку это сопротивление не бесконечно, между двумя проводниками возникает небольшой ток утечки. Этот ток утечки приводит к ослаблению (или ослаблению) сигнала, передаваемого по кабелю.

Проверьте свое понимание Сопротивление между двумя проводниками коаксиального кабеля зависит от удельного сопротивления материала, разделяющего два проводника, длины кабеля, а также внутреннего и внешнего радиуса двух проводников. Если вы разрабатываете коаксиальный кабель, как сопротивление между двумя проводниками зависит от этих переменных?

Чем больше длина, тем меньше сопротивление.Чем больше удельное сопротивление, тем выше сопротивление. Чем больше разница между внешним радиусом и внутренним радиусом, то есть чем больше соотношение между ними, тем больше сопротивление. Если вы пытаетесь максимизировать сопротивление, выбор значений для этих переменных будет зависеть от приложения. Например, если кабель должен быть гибким, выбор материалов может быть ограничен.

Исследование сопротивления провода — Курсовая работа по физике GCSE — Owlcation

Введение

В этой статье я исследую, что влияет на сопротивление провода.

Потоки электроэнергии в металлах. Металлические проволоки состоят из миллионов крошечных металлических кристаллов, и атомы каждого кристалла расположены в определенном порядке. Металл полон «свободных» электронов, которые не прилипают к какому-либо конкретному атому; скорее, они заполняют пространство между атомами. Когда эти электроны движутся, они создают электрический ток.

У проводников есть сопротивление, но некоторые из них хуже, чем другие. Свободные электроны продолжают сталкиваться с атомами. Сопротивление провода зависит от четырех основных факторов:

  • Удельное сопротивление
  • Длина провода
  • Площадь поперечного сечения
  • Температура проволоки

Я исследую, как длина провода влияет на сопротивление.Я провел предварительный эксперимент, чтобы помочь мне решить, как лучше всего провести расследование. Результаты также помогут мне делать прогнозы.

Предварительное расследование

Ниже приведены мои результаты предварительного эксперимента (см. Таблицу 1). Для обеспечения точности я снял по три измерения напряжения и силы тока.

Таблица 1: Предварительные результаты

Эти результаты показывают, что с увеличением длины провода увеличивается и сопротивление. Кроме того, если вы удвоите длину провода, сопротивление увеличится примерно вдвое.Например, при длине провода 20 см сопротивление составляет 3,14 Ом; когда длина провода 40 см, сопротивление составляет 6,18 Ом, что примерно вдвое больше. В моем основном исследовании я посмотрю, применимо ли это наблюдение к моим результатам.

Я обнаружил, что устройство, которое я использовал, было подходящим, но я думаю, что я мог бы увеличить количество точек данных для получения более надежных результатов, возможно, увеличивая длину провода на 5 см каждый раз, а не на 10 см.

Исследование сопротивления провода

Цель

Я исследую сопротивление провода в зависимости от его длины.

Прогноз

Я предсказываю, что чем длиннее провод, тем больше сопротивление. Это связано с тем, что свободные электроны в проводе сталкиваются с большим количеством атомов, тем самым затрудняя прохождение электричества. Точно так же, чем короче провод, тем меньше сопротивление, потому что электроны будут сталкиваться с меньшим количеством атомов, тем самым ослабляя поток электричества. Кроме того, сопротивление провода прямо пропорционально длине и обратно пропорционально площади, поэтому удвоение длины провода должно увеличить сопротивление в два раза.Это связано с тем, что если длина провода удвоится, электроны столкнутся с вдвое большим количеством атомов, поэтому сопротивление будет в два раза больше. Если это правильно, график должен показать положительную корреляцию.

Аппарат

Аппарат, который я буду использовать в этом эксперименте, выглядит следующим образом:

  • 1 амперметр (для измерения тока)
  • 1 вольтметр (для измерения напряжения)
  • 5 проводов
  • 2 зажима под кожу крокодила
  • Блок питания
  • нихромовая проволока 100см

Метод

Сначала я соберу необходимое мне устройство и настрою его, как показано на схеме 1 ниже. Затем я установлю блок питания на минимально возможное напряжение, чтобы гарантировать, что ток, проходящий через схему, не будет слишком высоким (что потенциально может повлиять на результаты, потому что провод станет слишком горячим).

Я размещу один зажим «крокодил» на расстоянии 0 см на проводе, а другой — на расстоянии 5 см, чтобы замкнуть цепь. Затем я включу блок питания и запишу показания вольтметра и амперметра. Я выключу блок питания, перемещу зажим «крокодил», который был на высоте 5 см, на 10 см, и включу блок питания.Опять же, запишу показания вольтметра и амперметра и выключу блок питания. Я буду повторять этот метод каждые 5 см, пока не доберусь до 100 см, снимая каждый раз по три показания с вольтметра и амперметра для обеспечения точности. Кроме того, после каждого считывания я выключаю блок питания, чтобы убедиться, что провод не слишком сильно нагревается и не влияет на мои результаты.

Диаграмма 1: Аппарат

Обеспечение точности

Для обеспечения точности я буду регистрировать напряжение и ток три раза каждые 5 см и снимать среднее значение. Это снизит вероятность ложных показаний и аннулирует любые аномальные результаты. Я также позабочусь о том, чтобы провод не слишком сильно нагрелся, подтвердив, что я не установил слишком высокое напряжение на блоке питания, и поддерживая то же напряжение при каждом показании. Кроме того, я обязательно выключаю блок питания после каждого чтения. Я постараюсь сделать это расследование максимально точным.

Переменные

В этом эксперименте можно изменять различные переменные; это независимая переменная.Однако в связи с моим запросом я изменю только длину провода. Переменные, которые я буду контролировать, будут типом провода (удельное сопротивление) и площадью поперечного сечения провода. Я также буду контролировать, используя блок питания, сколько вольт проходит через провод. Ниже приведена таблица, иллюстрирующая влияние изменения переменных (см. Таблицу 2):

Таблица 2: Переменные

Безопасность

Я обеспечу экспериментальную безопасность, убедившись, что все провода подключены правильно и что нет изоляции на проводах носится. Я также позабочусь о том, чтобы была четкая индикация того, что питание отключено с помощью переключателя и L.E.D. Я буду стоять во время расследования, чтобы не пораниться, если что-то сломается.

Результаты

Ниже представлена ​​таблица с моими результатами (Таблица 3). Я сделал три отсчета и вычислил среднее значение, показанное красным.

Таблица 3: Результаты

Таблица 4: Длина и сопротивление

Таблица 3 показывает, что с увеличением длины провода увеличивается и сопротивление.Это подтверждает первую часть моего прогноза: чем длиннее провод, тем больше сопротивление.

Кроме того, мое предсказание, что удвоение длины провода увеличивает сопротивление в два раза, верно (см. Таблицу 4).

График

График этих результатов показывает почти прямую линию, иллюстрирующую сильную положительную корреляцию между длиной и сопротивлением, что согласуется с моим прогнозом.

Обсуждение

В целом мои результаты полностью совпадают с моими прогнозами. Большинство точек данных находились на линии наилучшего соответствия или очень близко к ней. Есть несколько точек данных, которые находятся дальше от линии наилучшего соответствия, чем другие, но они все же согласуются с общей тенденцией. Нет никаких аномальных результатов, которые я бы считал далекими от линии наилучшего соответствия.

Существуют возможные источники ошибок, которые могли привести к противоречивым результатам, например изгиб провода. Это не позволило бы площади проволоки оставаться постоянной и повлияло бы на мои результаты.Однако я следил за тем, чтобы провод оставался прямым на протяжении всего эксперимента.

Я думаю, что диапазона моих результатов было достаточно, чтобы я мог сделать обоснованный вывод о том, как длина провода повлияла на сопротивление. Это произошло потому, что я мог построить график и показать общую тенденцию.

Я думаю, что закономерность / общая тенденция продолжится за пределами диапазона значений, которые я использовал. Однако я думаю, что, если бы у меня не было специального оборудования, результаты были бы искажены, потому что проволока со временем сильно нагрелась. Кроме того, аппарат, который я использовал в школе, не подошел бы, если бы я продолжал увеличивать длину провода; например, в классе я не мог увеличить длину более 150 см из-за соображений безопасности, а также из-за нехватки места.

Я думаю, что мой метод можно было улучшить, чтобы получить еще более стабильные результаты. Я мог бы подумать об использовании каждый раз нового отрезка проволоки, чтобы регулировать температуру более строго. Использование одного и того же куска проволоки на протяжении всего эксперимента означало, что его температура со временем немного повысилась, что могло повлиять на мои результаты.Однако использование новых кусков проволоки каждый раз было бы слишком непрактично и отнимало бы много времени в контексте этого урока. В целом, я считаю, что моего метода было достаточно для получения надежных результатов.

Чтобы подтвердить свое предположение и заключение, я мог бы провести дальнейшие эксперименты. Например, вместо нихрома я мог бы использовать разные типы проволоки. Я также мог бы рассмотреть возможность использования проводов с разным поперечным сечением или даже намеренного изменения температуры проводов и посмотреть, как манипулирование этими переменными влияет на сопротивление провода.

нина 30 марта 2020 г .:

оценка риска для этого?

Angkit Jeyachandran от 5 августа 2019 г .:

нуждается в оценке, плз

Натан 9 апреля 2019 г .:

Этот материал очень помог мне завершить мое физическое задание. большое спасибо

твоя мама гей 18 марта 2019 г .:

лол джомалл большой гей и собираешься вытащить деджи и проиграть 25 августа

как если ты согласен

игнорировать, чтобы умереть мгновенно

кимой смит от 12 марта 2019 г .:

Спасибо большое, это очень помогло мне и моим одноклассникам.

будет 4 марта 2019 г .:

очень хорошо, но где же зависимые и управляющие переменные? также включите некоторые экзаменационные вопросы и способы получения оценок, поскольку люди, которые это читают, с большей вероятностью будут сдавать экзамены! тем не менее, это очень хороший отчет об эксперименте.

Ria от 2 марта 2019 г .:

Большое спасибо, это действительно помогло мне с моим отчетом в лаборатории физики. Я обязательно размещу его на сайте и отдаю вам должное!

Гарри Патерсон 20 февраля 2019 г .:

Я люблю детей.

Jordan Fong 15 февраля 2019 г .:

Мне нравятся дети

hehohehohehoeh 7 января 2019 г .:

спасибо мне очень помогли

боб 7 декабря 2018 г .:

привет хорошие вещи

Flash от 6 декабря 2018 г .:

хорошие они мне действительно помогли

Lol 17 ноября 2018 г .:

всякая чушь неверная

Ур папа 15 ноября 2018 г .:

Я люблю ur willey

middsi 27 октября 2018 г .:

требует заключения, но это сделано очень хорошо.Спасибо. но также имейте в виду, что он не удваивается, а только увеличивается в небольших эпизодах xx

jamm 19 октября 2018 г . :

нуждается в оценке

Salal 13 июня 2018 г .:

Спасибо, это помогло Я был очень сбит с толку своими исследованиями по физике.

123456789 24 мая 2018 г .:

бесполезно

извините

lll 24 мая 2018 г .:

это было очень полезно для моей домашней работы, большое вам спасибо 🙂

Maya on 17 мая 2018 г .:

трата времени

Chicken Muncher 25 марта 2018 г .:

Вы ошиблись насчет удвоения длины провода = сопротивление увеличивается в два раза.

Я использовал некоторые данные для своего домашнего задания по физике, спасибо за это.

Просто не забывай жевать курицу, как я

ali 5 февраля 2018 г .:

Я скопировал и вставил все это для моего hw, и это было очень хорошо, спасибо

charlie houghton 22 января , 2018:

Большое спасибо, это действительно помогло мне понять этот эксперимент!

rio 27 ноября 2017 г . :

дал идеальные ответы, но для заключения вам нужно было добавить пример ваших результатов, это то, что говорит мой учитель естествознания, кроме того, это отличный источник информации, действительно помог я с моим домашним заданием

kcds 24 ноября 2017 г .:

вы только что выполнили мое задание: D спасибо, человек

bob 22 мая 2017 г .:

Хорошая работа.Молодец, Золотая Звезда. 🙂

angel kyeremeh 8 мая 2017 г .:

это так красиво. отлично сработано!

Тереза ​​ июня 25 апреля 2017 г .:

Какой у вас был SWG (стандартный калибр проволоки) или диаметр? Я знаю, что вы заявили, что это был нихром и длина 100 см, но SWG не указывается — я использую ваши данные как второстепенные, и мне нужно сравнить мое расследование и ваше.

сб? 13 апреля 2017 г .:

вы только что выполнили для меня задание спасибо boi

Retep882 30 марта 2017 г . :

Это, безусловно, самый зверский источник информации, который я смог найти для моей курсовой работы, вы классные 🙂

Мисс забывчивая. от 21 марта 2017 г .:

Большое спасибо за это, так как это очень помогло на моем уроке физики.

Эллиотт Хобноб 2 марта 2017 г .:

Привет, его Эллиот

Каллум Твизелл 27 февраля 2017 г .:

В моей концептуальной команде есть Ибра Месси и Саурес!

миша 03 февраля 2017 г .:

я вернулся

Привет 31 января 2017 г .:

Какие ссылки вы использовали?

yaas 31 января 2017 г .:

YAAASSS SLAY

MR Stone 27 января 2017 г .:

действительно полезно для моей оценки физики

Mike Litoris 27 января 2017 г .:

Забавно то, что я не девушка: /

Zac 23 января 2017 г .:

Меня отвлекли на все объявления

Биш Баш Бош k 16 января 2017 г . :

удивительно XDXDXDXDXDXDXDXD

Анонимный 7 января 2017 г .:

Какой диаметр или SWG используемого провода?

CAL на 20 октября 2016 г .:

ВЫ СОХРАНИЛИ МОЮ ЖИЗНЬ ДЛЯ МОИХ ВТОРИЧНЫХ ДАННЫХ БОЛЬШОЕ ОБРАЩЕНИЕ

gracealbam 17 октября 2016 г .:

Omg большое спасибо

9000o 13 L , 2016:

Love this lono 2012, what a geeza

XxX 06 сентября 2016:

мы можем быть вместе навсегда

doritoman 06 сентября 2016:

UR NOW EX

doritoman от 6 сентября 2016 г .:

I NO LUV U NO MO

doritoman 6 сентября 2016 г .:

Это было действительно полезно для моей контролируемой оценки.luv u

mehitsme 13 июля 2016 г .:

вы знаете, какая толщина провода? об этом здесь не говорится, и это была бы действительно хорошая информация для тех, кто, как я, должен использовать ее в качестве вторичного источника в своих контролируемых оценках.

Бен 8 февраля 2016 г .:

Спасибо, мне понадобился дополнительный источник данных для моей контролируемой оценки.

Dayanara 14 февраля 2015 г .:

Я очень доволен inootmarifn в этом.ТЫ!

Luno2012 (автор) из Великобритании, 20 марта 2014 г .:

@ehehfeelgood — курсовая работа предназначена для учащихся GCSE и средней школы.

ehehfeelgood 19 марта 2014 г .:

Спасибо, но это не подходит для тех, которые не умеют читать, знайте, должны быть картинки

helpmyballs 19 марта 2014 г .:

это так хорошо даже человек с синей бритвой использует его

BlueRazorBlade 19 марта 2014 г .:

Это очень помогло мне в моей научной работе y7 Спасибо

vikki 5 февраля 2014 г .:

Я ЛЮБЛЮ ТЕБЯ

Дарий 17 декабря 2013 г .:

Спасибо! Это действительно помогло мне с работой в 9 классе. Я очень ценю информацию, которую вы ввели в это. Еще раз спасибо!

omt8 7 ноября 2013 г .:

Отличная помощь. Мне очень помогли мои курсовые работы в 9 классе. спасибо

Luno2012 (автор) из Соединенного Королевства 26 октября 2013 г .:

Сэм: Не беспокойтесь.

MrsBonnersSciencePupil: Рад, что смог помочь!

Люси: Рада это слышать, Люси. Удачи вам с будущими тестами и экзаменами.

Люси 13 октября 2013 г .:

привет, это было действительно полезно, я сейчас занимаюсь CAU за 11 год, и это было большим подспорьем, спасибо

MrsBonnersSciencePupil 18 июня 2013 г .:

Это было действительно полезно для моей контролируемой оценки.luv u

sam waiganjo kenya от 4 октября 2012 г .:

тыс. много

Resolver2009 из Борнмута, Великобритания / Осло, Норвегия 27 апреля 2012 г .:

Проголосовали 🙂 Должно быть очень полезно для студентов, изучающих физику.

Цель — 8: Сопротивление металлического проводника — CCEA — Редакция GCSE Physics (Single Science) — CCEA

Экспериментально исследовать, как сопротивление металлического проводника при постоянной температуре зависит от длины, и получить достаточные значения для построения графика сопротивления (ось y) и длины (ось x).

Переменные

Основными переменными в научном эксперименте являются независимая переменная, зависимая переменная и контрольные переменные.

Независимая переменная — это то, что мы изменяем или контролируем в эксперименте.

Зависимая переменная — это то, что мы тестируем и будем измерять в эксперименте.

5q7uy1xsig.0.0.0.1:0.1.0.$0.$1.$6″> Контрольные переменные — это то, что мы сохраняем неизменными во время эксперимента, чтобы убедиться, что это справедливый тест.

В этом эксперименте:

  • Независимая переменная — это длина провода.
  • Зависимая переменная — это сопротивление провода.
  • Управляющие переменные — это материал, площадь поперечного сечения и температура провода. Они сохраняются неизменными за счет того, что не меняют провод во время эксперимента, сохраняют небольшой ток и открывают переключение между показаниями.

Помните — эти переменные контролируются (или остаются неизменными), потому что для проверки достоверности можно изменить только одну переменную, которая в данном случае является длиной провода.

Уравнение

Сопротивление R = \ (\ frac {напряжение ~ V} {ток ~ I} \)

Прогноз

По мере увеличения длины провода сопротивление будет расти.

Обоснование прогноза

Чем больше длина провода, тем больше количество столкновений между свободными электронами и ионами металлов.

Это приведет к большему сопротивлению.

Безопасность

Опасность Последствия Меры контроля
$18.$2.$0.$0″> Вода Поражение электрическим током Не ставьте эксперимент рядом с кранами, раковинами и т. Д.
Проволока нагревается Незначительные ожоги Не трогайте провод. Выключите между измерениями.

Аппарат

Длина константанового провода 1 м, линейка для счетчика, блок питания низкого напряжения, вольтметр, амперметр, соединительные провода, выключатель, 2 зажима типа «крокодил», скотч.

Метод

  1. Настройте схему, как показано выше. Прикрепите гибкий провод на отметке 20 см, чтобы длина провода, по которому протекает ток, составляла 20 см.Запишите эту длину в подходящую таблицу.
  2. Отрегулируйте блок питания, пока ток на амперметре не станет 0,4 А. Запишите значение тока в таблицу.
  3. Считайте соответствующее значение напряжения на проводе на вольтметре и запишите в таблицу.
  4. Выключите выключатель, чтобы предотвратить повышение температуры проволоки.
  5. Включите снова и повторите измерение напряжения. Запишите в таблицу. Выключите и рассчитайте среднее напряжение.
  6. Рассчитайте сопротивление этой длины провода и запишите его в таблицу.
  7. Включите снова. Убедитесь, что ток по-прежнему составляет 0,4 А, и повторите считывание тока и напряжения для длин 40 см, 50 см, 60 см, 80 см и 100 см.
  8. Рассчитайте сопротивление для каждой длины, не забывая выключать между каждым считыванием.

Ошибка

Температура провода должна поддерживаться постоянной.

Когда через проводник течет ток, возникает эффект нагрева.

$0.$1.$27″> Электрическая энергия преобразуется в тепловую.

Чтобы температура провода не повышалась, в перерывах между снятием показаний выключайте ток и поддерживайте минимальный ток.

Точно снимите показания амперметра и вольтметра, считывая шкалу непосредственно над указателем, или используйте цифровые приборы.

Удельное электрическое сопротивление | Основы резистора

Что такое удельное электрическое сопротивление?

Удельное электрическое сопротивление — это мера способности материала препятствовать прохождению электрического тока. Выражается в Ом-метрах (Ом⋅м).Символом удельного сопротивления обычно является греческая буква ρ (ро). Высокое удельное сопротивление означает, что материал плохо проводит электрический заряд.

Удельное электрическое сопротивление определяется как отношение между электрическим полем внутри материала и электрическим током, проходящим через него, как следствие:

, где ρ — удельное сопротивление материала (Ом · м),

E — величина электрического поля в материале (В / м),

Дж — величина плотности электрического тока в материале (А / м 2 )

Если электрическое поле ( E ) через материал очень велико, а протекание тока ( Дж ) очень мало, это означает, что материал имеет высокое удельное сопротивление.

Электропроводность — это инверсия удельного сопротивления и мера того, насколько хорошо материал проводит электрический ток:

, в котором σ — проводимость материала, выраженная в Сименсах на метр (См / м). В электротехнике часто вместо σ используется κ (каппа).

Электрическое сопротивление

Электрическое сопротивление выражается в Ом и не совпадает с удельным сопротивлением. В то время как удельное сопротивление — это свойство материала, сопротивление — это свойство объекта.Электрическое сопротивление резистора определяется сочетанием формы и удельного сопротивления материала. Например, резистор с проволочной обмоткой с длинным и толстым проводом имеет более высокое сопротивление, чем с более коротким и тонким проводом. Резистор с проволочной обмоткой, изготовленный из материала с высоким удельным сопротивлением, имеет более высокое значение сопротивления, чем резистор с низким удельным сопротивлением. Можно провести аналогию с гидравлической системой, где вода перекачивается по трубе. Чем длиннее и тоньше труба, тем выше будет сопротивление.Труба, заполненная песком, будет противостоять потоку воды больше, чем труба без песка (свойство удельного сопротивления).

Гидравлическая аналогия электрического сопротивления

Сопротивление провода

Значение сопротивления провода зависит от трех параметров: удельного сопротивления, длины и диаметра. Формула для расчета сопротивления провода следующая:

, в котором R — сопротивление (Ом)

,

ρ — удельное сопротивление материала (Ом · м),

l — длина материала (м),

A — площадь поперечного сечения материала (м 2 )

Значение сопротивления провода зависит от трех параметров; его удельное сопротивление, площадь поперечного сечения и длина.

В качестве примера рассмотрим проволочный резистор с проволокой из нихрома с удельным сопротивлением 1,10 × 10 –6 Ом · м. Проволока имеет длину 1500 мм и диаметр 0,05 мм. С помощью этих трех параметров рассчитывается значение сопротивления:

Нихром и константан часто используются в качестве проволоки сопротивления. Посмотрите в таблице удельное сопротивление материалов для часто используемых материалов.

Сопротивление листа

Значение сопротивления листа рассчитывается точно так же, как и сопротивление провода.Площадь поперечного сечения может быть записана как произведение w и t :

.

Для некоторых применений, таких как тонкие пленки, соотношение между удельным сопротивлением и толщиной пленки называется сопротивлением листа R s :

, в котором рупии выражены в омах. Для этого расчета толщина пленки должна быть одинаковой.

Часто производители резисторов увеличивают сопротивление, вырезая узор на пленке, чтобы увеличить путь электрического тока.

Электрическое сопротивление листа зависит от длины, ширины, толщины пленки и удельного сопротивления. Сопротивление можно увеличить, вырезав узор на листе.

Резистивные свойства материалов

Удельное сопротивление материала зависит от температуры и обычно дается для комнатной температуры (20 ° C). Изменение удельного сопротивления в результате изменения температуры описывается температурным коэффициентом. Например, термисторы используют это свойство для измерения температуры.С другой стороны, в прецизионной электронике это обычно нежелательный эффект. Резисторы из металлической фольги обладают отличными свойствами в отношении температурной стабильности. Это достигается не только за счет низкого удельного сопротивления материала, но и за счет механической конструкции компонента.

Для изготовления резисторов используется множество различных материалов и сплавов. Нихром, сплав никеля и хрома, часто используется в качестве материала для проволоки резистора из-за его высокого удельного сопротивления и не окисляется при высоких температурах.-3 Серебро 1,59 × 10 −8 6,30 × 10 7 3,8 Медь 1,68 × 10 −8 5,96 × 10 7 3,9 Золото 2,44 × 10 −8 4,10 × 10 7 3,4 Алюминий 2,82 × 10 −8 3,5 × 10 7 3.9 Вольфрам 5,60 × 10 −8 1,79 × 10 7 4,5 цинк 5,90 × 10 −8 1,69 × 10 7 3,7 Никель 6,99 × 10 −8 1,43 × 10 7 6 Литий 9,28 × 10 −8 1,08 × 10 7 6 Утюг 1.0 × 10 −7 1,00 × 10 7 5 Платина 1,06 × 10 −7 9,43 × 10 6 3,9 Олово 1,09 × 10 −7 9,17 × 10 6 4,5 Свинец 2,2 × 10 −7 4,55 × 10 6 3,9 Манганин 4.82 × 10 −7 2,07 × 10 6 0,002 Константан 4,9 × 10 −7 2,04 × 10 6 0,008 Меркурий 9,8 × 10 −7 1,02 × 10 6 0,9 нихром 1,10 × 10 −6 9,09 × 10 5 0,4 Углерод (аморфный) 5 × 10 −4 до 8 × 10 −4 1.От 25 до 2 × 10 3 -0,5

Электрическое сопротивление и класс сопротивления десять по науке NCERT


Сопротивление

Сопротивление — это свойство проводника, благодаря которому он сопротивляется прохождению через него электрического тока. Компонент, который используется для сопротивления прохождению электрического тока в цепи, называется резистором.

На практике резисторы используются для увеличения или уменьшения электрического тока.

Переменное сопротивление: Компонент электрической цепи, который используется для регулирования тока; без изменения напряжения от источника; называется переменным сопротивлением.

Реостат: Это устройство, которое используется в цепи для обеспечения переменного сопротивления.


Причина сопротивления в проводнике:

Поток электронов в проводнике — электрический ток. Частицы проводника создают препятствие потоку электронов; из-за притяжения между ними.Эта помеха является причиной сопротивления току электричества.

Сопротивление проводника зависит от типа, длины и площади поперечного сечения проводника.

Тип материала: Некоторые материалы создают наименьшие помехи и поэтому называются хорошими проводниками. Серебро — лучший проводник электричества. В то время как некоторые другие материалы создают больше препятствий для прохождения электрического тока, то есть прохождения через них электронов. Такие материалы называют плохими проводниками.Плохие проводники также известны как изоляторы. Жесткий пластик — один из лучших изоляторов электричества.

Длина проводника: Сопротивление R прямо пропорционально длине проводника. Это означает, что сопротивление увеличивается с увеличением длины проводника. Это причина того, что длинные электрические провода создают большее сопротивление электрическому току.

Таким образом, сопротивление (R) ∝ длина проводника (l)

или `R ∝ l` ——— (i)

Площадь поперечного сечения: сопротивление R обратно пропорционально площади поперечного сечения (A) проводника.Это означает, что R будет уменьшаться с увеличением площади проводника и наоборот. Большая площадь проводника облегчает прохождение электрического тока через большую площадь и, таким образом, снижает сопротивление. Это причина того, что толстый медный провод создает меньшее сопротивление электрическому току.

Таким образом, `текст (сопротивление) ∝ (1) / (текст (площадь))` поперечного сечения проводника (A)

Или, `R∝1 / A` ——— (ii)

Из уравнений (i) и (ii)

`R∝l / A`

Или, `R = ρ \ l / A` ———— (iii)

Где ρ (rho) — константа пропорциональности.2) / (м) = & Ом; м`

Таким образом, единицей измерения удельного сопротивления (ρ) в системе СИ является Ом · м

Материалы с удельным сопротивлением в диапазоне от 10 -8 Ом · м до 10 -6 Ом · м считаются очень хорошими проводниками. Серебро имеет удельное сопротивление, равное 1,60 X 10 −8 Ом · м, а медь имеет удельное сопротивление 1,62 · 10 −8 Ом · м.

Резина и стекло — очень хорошие изоляторы. Они имеют удельное сопротивление порядка от 10 12 Ом · м до 10 17 Ом · м.2) `

`= (10xx2 & ohm;) / (7.65xx10)`

`= (2 Ом;) / (7,65) = 0,26 Ом;`

Пример 4: Площадь поперечного сечения провода уменьшается вдвое, когда его длина увеличивается вдвое. Как изменится сопротивление провода в новом состоянии?

Решение: Пусть площадь сечения провода =

А

Пусть длина провода перед натяжкой = L

Пусть Сопротивление провода =

R

После натяжения проволоки пустить

Площадь поперечного сечения = A / 2

Длина = 2L

Сопротивление = R1

Таким образом, отношение сопротивления до растяжения к сопротивлению после растяжения можно представить следующим образом:

Или, `R: R_1 = (ρL) / (A): (ρ2L) / (A / 2)`

Или, `R: R_1 = (ρL) / (A): (4ρ L) / (A)`

Или `R: R_1 = 1: 4`

Это означает, что R = 1 и R1 = 4

Таким образом, после растяжения проволоки сопротивление увеличивается в четыре раза.

Провод

Добавить комментарий

Ваш адрес email не будет опубликован. Обязательные поля помечены *